HEENT: Vignette Questions

¡Supera tus tareas y exámenes ahora con Quizwiz!

Benign Paroxysmal Positional Vertigo diagnose by Dix hallpike and treat with Epley manuever ****tympanogram shoulder be normal****

29 yo F presents with recurrent "dizziness" for the past few days that worsens with movement. She states her episodes last 1-2 minutes and denies hearing loss, tinnitus, or hx of headaches. How would you diagnose and treat? Likely diff dx?

Papilloma Sx

34 y/o M presents for the following painless, but rapidly growing growth.

Laryngeal Leukoplakia Laryngoscopy w bx Dc smoking PPI therapy Radiation

66 y/o smoker referred to ENT for unexplained hoarseness.

Your patient is 54-year-old immunocompetent male that presents with right-sided parotitis, trismus, and a fever of 101.6. Which of the following antibiotic regimens are most appropriate? a. Naficillin and Metronidazole b. Naficillin c. Metronidazole d. Azithromycin

A - Empiric therapy for parotitis involves nafcillin and metronidazole. Vancomycin and cefepime or imipenem is recommend for immunocompromised patients. Azithromycin does not cover staph.

sensorineural hearing loss

A 32 yo M presents with worsening hearing loss. He reports that he is a musician and is around loud music daily. What do you have concern for when he has better air conduction upon Rinne exam?

is characterized by a pale retina, as well as a cherry red spot on the macula.

A central retinal artery occlusion

Traumatic hearing loss possibly due to a skull fx

A patient presents s/p recent MVA of whom is complaining of slight difficulty hearing since the accident. What is your major concern?

How would you treat a patient with allergic rhinitis?

Avoid allergen Steroid nasal spray Saline irrigation

In infants, the eyes should move in parallel without deviation by the age of A. 2 weeks B. 3 months C. 6 months D. 1 year

C. 6 months

How would you treat a patient with epistaxis?

Direct pressure for 10-15 minutes while leaning forward Afrin Packing

How would you treat a patient with a septal perforation?

ENT referral Saline irrigation Humidifier

How would you treat a patient is fungal sinusitis?

ENT referral for surgical debridement Saline irrigation Nasal and systemic steroids

Your patient is 64 year old female that presents with sudden onset of right eye pain, redness, and photophobia with tearing. Slit lamp exam reveals some conjunctival erythema and a shallow anterior chamber. There is no ciliary flair. The intraocular pressure is 44 in the right eye. All of the following are acceptable management options except? a. referral to ophthalmology b. begin Xaltan drops c. instill cycloplegic drops d. start norco as needed

NOTICE THE WORD EXCEPT!!! C - This patient has acute closure glaucoma. Installation of a cycloplegic can increase the pressure and make the patient worse. A cycloplegic drop causes paralysis of the ciliary muscle of the eye, resulting in a loss of accommodation and flexibility; Xalatan drops = help to lower IOP; definitely going to want to refer to ophthalmology; Norco = use for pain

a 46-year-old male with an involuntary, rapid, and repetitive movement of both eyes side to side. Dx Tx

Nystagmus -symptomatic: Surgery

A 7 year old boy is brought in complaining of ear pain. On exam you note a white discharge, pruritis, pain and tenderness of the tragus. The weber test lateralizes bilaterally, and the Rhine test has AC>BC bilaterally. DX? TX?

Otitis Externa -Antibiotic drops: ofloxacin -otowick -avoid water -Drying agents -No Q-tips

How would you treat a patient with Rhinitis medicamentosa?

STOP AFRIN

A 68-year-old man presents to the outpatient clinic complaining of decreased hearing in his left ear. The following is seen on otoscopic evaluation. Which of the following is the most likely diagnosis? A. Acute otitis media B. Cerumen impaction C. Cholesteatoma D. Chronic otitis media

The answer is C. EXPLANATION: The photo depicts a classic cholesteatoma (C) effecting the pars flaccida. The localized nature of the findings and lack of inflammation make otitis media (A and D) unlikely. Any perforation in the TM (E) would be secondary to the cholesteatoma and the visible TM is inconsistent with a diagnosis of impacted cerumen (B).

Which of the following conditions is a cause for central vertigo? A. Meniere syndrome B. Labyrinthitis C. Vestibular neuronitis D. Acoutic neuroma E. Perilymphatic fistula

The answer is D. EXPLANATION: Acoustic neuroma, or eight cranial nerve schwannomas, are among the most common intracranial tumors, and a cause for central vertigo.

Unilateral hearing loss is suggests peripheral vertigo. The exception is vestibular schwannoma which is a central lesion as most lesions are unilateral which causes unilateral hearing loss.

What does unilateral hearing loss mean in regarding to vertigo? What is the exception?

manage DM laser photocoagulation

What is the appropriate tx plan for non proliferative diabetic retinopathy?

Herpes Zoster ophthalmicus treatment

Zostavax (>50)

9 y/o M presents to pediatrician w/ mother c/o cough, sore throat, fever, diarrhea and runny nose. Which of the following treatments would you suspect is appropriate for the dx? a. ABX b. hydrate/lozanges c. IM steroid injection d. NSAIDs

b. hydrate/lozanges

Which of the following is not a recommended treatment for macular degeneration? a. monoclonal ab therapy b. laser tx c. surgery d. antiVEGE

c. surgery

Corneal Abrasion symtoms

- injected conjunctivitis -Swollen lids -Corneal defect

Open Angled glaucoma symptoms

- insidious, symptomatic rise in IOP -Asymptomatic until late -Chronic &slow progressive vision loss -Increase cup to disc ratio

Retinal Artery Occlusion treatment

-High flow O2 -Intermittent firm pressure massage -Referral for vessel dilation & paracentesis

Pinguecula Artificial tears

89 y/o M presents w the following. Has not grown into the cornea

Your patient is a 45 year old male that is not immunized. He presents with a fever of 102, dysphagia, drooling, and shortness of breath. Lungs are clear. O2 Sat 100%. Pulse 102. RR-24. Soft tissue neck reveals a thumb sign. All of the following are acceptable management plans except: a. give a Racemic Epinephrine aerosol treatment and discharge the patient home on steroids b. begin Vancomycin and Ceftriaxone c. intubate the patient in the operation room d. consult anesthesia or ENT

A - (Epiglottis) B-D is acceptable (first you should intubate the pt and D at the same time, then B). A isn't gonna help at all

Your patient is a 32 year-old female that presents with a nodular rubbery lesion to her upper eyelid. It is not painful. It has been there for 6 days. Which of the following is the most likely diagnosis? a. Chalazion b. Hordeolum c. Dacryocystitis d. Foreign Body e. Ectropion

A - A chalazion is an inflammatory lesion that developed when the meibomian tear gland becomes obstructed. They are usually not painful. Treatment is warm compresses or surgical correction. - Hordeolum: usually painful - Foreign body: nothing to prove this - Dacryocystitis: inflammation of the nasolacrimal sac from an obstruction of the nasolacrimal system; lower lid is affected; no lesions - Ectropion: lower lid droops (no lesions)

ETD tinnitus Treatment is directed to ETD which is beclomethasone nasal spray and decongestants.

A 29 yo F presents with ringing in her ears that began 3 days ago. She reports recently feeling a fullness to her ears that is uncomfortable and the ringing seems to be in tune her how fast she breathes. What is her likely dx and treatment?

The patient has an aphthous ulcer likely due to Recurrent Aphthous Stomatitis due to history of celiacs. Treatment is good oral and valcyclovir.

A 29 yo M with hx of celiacs disease presents to the clinic with a oval shaped oral ulcer to the buccal mucosa. Patient states that he has been very stressed recently and has a history of similar sxs. What is his diagnosis, treatment, and likely cause?

Must image in order to rule out squamous cell laryngeal cancer. If cancerous must consider a type of laryngectomy.

A 43 yo F smoker presents to the clinic with a hoarse voice that has been ongoing for 2 months. Patient reports that she otherwise feels fine and has no pain associated with her hoarseness. What is a diagnostic option and your differential dx?

Perform a carotid US due to concern for neuro (pulsatile) tinnitius

A 43 yo M presents with ringing in his ear that began 3 months ago. His exam shows a LEFT carotid bruit. What diagnostic test do you perform and what is your diff dx?

Due to a history of frequent strep, the patient could have Ludwig's Angina where the most common causative agent is Strep Viridans. The major initial concern is airway management then can treat with IV amoxicillin depending on severity.

A 45yo F presents with a history of a dental abscess that never completely resolved complaining of unilateral sore throat that extends to her neck. The patient reports frequent strep infections as well and you note she is speaking with a "hot potato" voice. What is your major concern, likely diagnosis, and causative pathogen?

D Ménière's syndrome Ménière's syndrome usually presents with episodes of vertigo that last from 1 to 8 hours, sensorineural hearing loss and tinnitus.

A 52 year-old female presents with complaints of intermittent episodes of dizziness, tinnitus, and hearing loss in the right ear for 6 months. She describes the dizziness as the "room spinning around her," with the episodes typically lasting for 2 to 4 hours. Physical examination reveals horizontal nystagmus and right ear hearing loss, but the remainder of the examination is unremarkable. Which of the following is the most likely diagnosis? A Acute labyrinthitis B Positional vertigo C Acoustic neuroma D Ménière's syndrome

Likely a corneal abrasion which is a effect of chronic entropion involution. Might have to perform a fluorescein stain to identify. Can treat with botox in order to relax eyelid.

A 65 yo F presents with ongoing entropion and a FB sensation. Patient denies any vision changes but has had irritation to the eye. What is your diff dx and first step in order to diagnose?

Open angle chronic glaucoma treat with prostaglandin eye drops

A 69 yo M presents with ongoing "tunnel vision" without any painful vision loss. You know that his exam has optic cupping and a slightly elevated IOP. What is his likely diagnosis and treatment options?

Atrophic Oral candidiasis or denture stomatitis Treatment is Nystatin swish and swallow and clotrimazole (if HIV negative) *Also mention that longterm use of nystatin can cause dental caries.

A 73 yo M presents to the clinic with painful red spots to his oral mucosa. Exam shows no plaques or lesions to the mucosa but you note that the patient had to remove their dentures prior to exam. What is his diagnosis and treatment?

A 33 year-old presents with sinusitis unresponsive to three various antibiotics over the past four months. Sinus puncture is performed with culture positive for Aspergillus fumigatus. What is the most appropriate treatment for this patient? A Amphotericin (Amphotericin B) B Amoxicillin/Clavulanate (Augmentin) C Ofloxacin (Floxin) D Cefuroxime (Ceftin)

A. Amphotericin (Amphotericin B)

A patient presents with complaint of sudden onset of recurrent episodic vertigo for one week that happens when rolling onto the left side. The patient states that this sensation lasts approximately 30 seconds and then goes away. The patient admits to associated nausea. The patient denies associated hearing difficulties or tinnitus. Which of the following is the most likely diagnosis? A Benign positional vertigo B Ménière's disease C Acoustic neuroma D Vestibular neuronitis

A. Benign positional vertigo

A 64 year-old woman complains of headache and left eye pain for about a day. She says it started yesterday as a dull ache and now is throbbing. She also complains of nausea and vomiting, which she attributes to the popcorn she ate at the movie theater yesterday afternoon. On exam, the left pupil is mid-dilated and nonreactive. The cornea is hazy. A ciliary flush is noted. Which of the following is the most likely diagnosis?

Acute angle glaucoma

Which of the following is the most common etiologic agent associated with acute bacterial sinusitis in the adult population? A Staphylococcus aureus B Streptococcus pneumoniae C Pseudomonas aeruginosa D Mycoplasma pneumoniae

B. Streptococcus pneumoniae

A 23 year-old graduate student presents with sudden onset of severe dizziness, with nausea and vomiting for the past couple of hours. She denies hearing loss or tinnitus. She has had a recent cold. Which of the following is the most likely diagnosis? A Ménière's disease B Vestibular neuronitis C Benign positional vertigo D Vertebrobasilar insufficiency

B. Vestibular neuronitis

Which of the following is not true regarding entropion? a. It is a turning in of the edge of an eyelid b. artificial tears help with dryness c. trachoma infection can cause lower lid infection d. usually affects the upper eyelid

D - usually affects the lower eyelid

Meniere's disease

Endolymphatic hydrops​ → ​episodic increased inner ear pressure

a 75-year-old with a foreign body sensation and tearing of his right eye. On physical exam you note a red, irritated, right eye in association with an inverted eyelid. Dx? TX?

Entropion -Eye lubricating drops -Surgery

3-year-old who is brought into the emergency room by her parents. The child has had a high fever, sore throat, and stridor. She has a muffled voice and is sitting up on the stretcher drooling while leaning forward with her neck extended. The patient's parents are adamantly against vaccinations, claiming that they are a "government conspiracy." You order a lateral neck x-ray, which shows a thumbprint sign. Dx: Pathogen: Tx:

Epiglottitis -H. Flu -Immediate ENT consult -Don't change child's position -O2 sat & secure airway -ICU -IV abx + steroid ceftrizoxime or cefuroxime + dexamethasone (slides) - prednisone & ceftriaxone. (Smartypance)

a 17-year-old on the high school varsity wrestling team was injured during a match. On physical exam, you note a fluctuant, tender edematous lesion of the anterior-superior outer portion of the right pinna Dx? Tx?

External Ear trauma - I&D (hematoma) -Refer to plastics

Patient presents with tender frontal and maxillary sinuses. They state that their mucous discharge is dark and rubbery.

Fungal sinusitis

________ __ and _____________ can lead to nerve palsies.

Malignant OE and Cholesteatoma

a 4-year-old girl who is brought to the clinic by her mother who states that the child has been complaining of progressively worsening ear pain and itchiness over the past week. Examination reveals left tragal tenderness and an edematous and closed canal. Weber lateralizes to the left. Dx: Pathogen: Tx:

Otitis Externa - Pseudomonas aeruginosa - Antibiotic drops: ofloxacin -Otowick -Drying agents -Avoid water -Stop Q-tip use

Hordeolum Warm compresses

Painful, swollen, warm red lump.

Dental abscess I+D Abx

Pt presents for pain + tenderness of the upper middle teeth.

Patient presents with nasal obstruction. Physical exam shows bilateral septal asymmetry with mucosal swelling. They look like small grapes.

Septal hematoma

Herpes Zoster ophthalmicus Complications

Uveitis, glaucoma, scleritis optic neuritis

All keratitis have this in common

Vision changes photophobia red eyes tearing may need a corneal transplant if they are super severe

Which of the following can shrink the size of nasal polyps? a. omalizumab b. azestelin c. glucocorticoid nasal spray d. antileukotriene therapy

a. omalizumab

Which patient population is most susceptible to cholesteatomas? a. 6 mo olds b. Turner syndrome c. SLE d. Deaf

b. Turner syndrome

78 y/o M presents to PCP w/ spinning sensation, N/V especially when laying down in bed. PT denies any ear pain, hearing loss or tinnitus. You suspect BPPV. How do you treat this patient and manage the sx? a. D/c current medications b. Epley maneuver c. Antivert/Meclizine + Zofran d. Trimethopri-sulfamethorazole

c. Antivert/Meclizine + Zofran

Mononucleosis is caused by: a. Group A strep b. Gonorrhea c. EBV d. h. influenza

c. EBV

chronic HTN

optic disc swelling and abnormal light reflexes

A man comes into the ER and can hardly see out of his left eye. He has double vision and is bleeding everywhere. A closer look reveals a staple has gotten caught in his eye. Which test will be positive? What will the RR be like? What about the pupil? What is the treatment for globe rupture?

+ Siedel's test Obscured RR prolapse of iris thru cornea rigid eye shield Leave impaled object +/- tetanus ppx Avoid topical solutions/pressure

Closed Angle Glaucoma symptoms & tx

- EMERGENCY -Ocular pain & dec vision -Halos around light -Injected conjunctiva Reduces visual acuity -Pupil is mildly reactive in a mid-dilated state N/abd pain _IOP >40 -Visual field defects -enlarged/ excaved optic disc with pallor -Tx: Laser (dec IPO) -Systemic acetazolamide, optic dirertics, pilocarpine -AVOID Anticholinergics

A 37-year-old woman comes to your office for assessment of hearing loss. She has had problems intermittently for the past 12 months. On examination, the Weber tuning fork test lateralizes to the right ear, and the Rinne tuning fork test is negative in the right ear (bone conduction is greater than air conduction [BC > AC]). This suggests which of the following? A. A right-sided conductive hearing loss B. A left-sided conductive hearing loss C. A right-sided sensorineural hearing loss D. A left-sided sensorineural hearing loss

- Weber lateralizes to the affected ear; Rinne = AC > BC—SNHL; BC > AC—CDHL - in this example Weber = lateralizes to right ear — affected ear; Rinne = right ear BC > AC — CDHL, therefore, A is the correct answer

Symptoms of Herpes Simplex Keratitis

- irritation, Light sensitivity, redness - May have pain -Dendritic lesions -scaring, vascularization

Retinal Artery Occlusion symptoms

-Acute -Painless -Unilateral vison loss on exam: -Boxcarring -opalescent retina, cherry red spot -optic atrophy, pale disc, blind eye

What are the ddx for horizontal nystagmus, sudden tinnitus and HL? what is the category? What does it affect?

-BPPV MC -Meniere's -Vestibular Neuritis -Labyrinthitis -Cholesteatoma Peripheral Vertigo labyrinth or vestibular nerve

A 70 year old female comes into the office complaining of unilateral vision loss. The patient states that is happened suddenly, but it is not painful. On exam you note boxcarring, opalescent retina, a pale disc, optic atrophy and a cherry red spot. What is the appropriate treatment & dx?

-Central Retinal Artery occlusion -High flow O2 -Intermittent firm pressure -Referral

Symptoms of Amaurosis Fugax

-Curtain descends -Transient acute loss of vision -Unilateral vision loss that last several minutes -TIA criteria

HTN Retinopathy Symptoms

-Diffuse arteriolar narrowing -optic disc edema

Macular Degeneration symptoms

-Drusen deposits -Insidious onset -Gradual vision loss Metamorphsia (asmler grid) -Mottling, serous leaks, & hemorrhages in retina

an 18-month-old with ear pain and otorrhea. Otoscopic examination reveals a small insect impacted in the ear canal which is still moving. The tympanic membrane appears intact. Dx? Tx?

-External Auditory Canal Foreign Body - Solid: a loop, alligator forcepts -Organic: NO IRRIGATION -Insect: Lidocaine/ mineral oil -Irrigation (if TM intact) -Topical abx if needed

Central Retinal Vein occlusion Treatment

-May resolve spontaneously -Laser tx -Workup underlying cause

A 50 y/o female with a PMH of MS comes in complaining of sudden L eye vision loss. The patient has pain with extraocular movement. and on exam you note that when you shine a light in the L eye both eye slightly dilate, when you shine a light in the R eye both eye constrict. What is the appropriate dx and tx?

-Optic Neuritis -treat MS -IV corticosteroids

a 2-year-old who arrives at the ED with a swollen and erythematous eyelid, proptosis, pain with movement of the eye, and an inability to adduct or abduct his eye. DX? TX?

-Orbital cellulitis -Broad spectrum abx IV then PO - Ampicillin-sulbactam, cephalosporin, chloramphenicol -Monitor -surgical drainage if lg Sinus drainage if indicated

Herpes Zoster ophthamicus symptoms

-Pain, headache, photophabia, malaise, fevere -Hutchinson's sign (lesion on tip of nose)

Corneal Ulcer symptoms

-Pain, photophabia, tearing, reduced vison -Dense corneal infiltrate -Ciliary flush -Rapid corneal destruction (severe) -Feathery Boarder (fungal)

Corneal Ulcer tx

-Refer -Avoid contacts -Ofloxacin, ciprofloxacin, erythromycin (SMARYPANCE)

Corneal Abrasion tx

-Refer if deep -Topical abx (oflaxacin, -educate on safaty glasses use

Retinal detachment Treatment

-Referral to ophthalmology -Pt position (side of detachment to pillow - L vision loss, lye on R side) - Cryotherapy/ laser -surgery

A 44 year old male comes into your office today complaining of sudden peripheral vision loss, photopsia, and seeing shadows. On HEENT exam you note a asymmetric red reflex. What is the dx, and treatment plan?

-Retinal detachment -Position the patient to lye down on effected side. -Referral to ophthalmology -Cryotherapy

Retinal Detachment Symptoms

-Spontaneous -Photopsia -Progressive(central vision, spared till macular detaches) -Asymmetric red reflex

Central Retinal Vein occlusion symptoms

-Subacute unilateral blurriness/ VL -Afferent pupillary defect -Blood & Thunder fundus -Neovascularization

Optic Neuritis Symptoms:

-Unilateral, sudden vision loss -Loss of pupillary rxn to light -Pain with extraocular movements Afferent pupillary defect (Marcus Gunn Pupil) -Papilledema

Optic Neuritis TX

-tx underlying cause -IV corticosteroids -Good prognosis (unless SLE, MS)

Acute pharyngitis (bacterial- GAS + anterior LAD. viral - posterior LAD) RADT/ throat swab

10 y/o M presents for sore throat + pain w swallowing. PE + for anterior LAD and white-yellow exudate.

Asymmetric tonsils can be an early sign of lymphoma and that a biopsy might be a good next step.

14 M presents to clinic with his mother of whom is concerned that her child has tonsilitis. His exam show asymmetric tonsils. You tell the mother that it is not infected but there is concern for what?

Mucocele Excision of lesion and minor salivary gland

15 y/o F presents for the following after getting hit in the face with a soccer ball. Mass has changed in size since she was hit a couple days ago.

OE Acute = topical corticosteroids or topical abx (cipro) Chronic = topical steroids

16 y/o F presents for R ear pain. PE + R ear TTP and manipulation of the tragus; edema of the ear canal. No PMHx. Pt swims competitively.

Hematoma of external ear Refer to ENT for drainage ASAP

16 y/o wrestler presents for tender lesion on his left ear.

EBV + Ab test, throat swab Symptomatic

19 y/o F presents for fever, fatigue, posterior chain LAD and splenomegaly.

Tracheomalacia XR Obs and CPAP

2 y/o F presents for WCC. PE + noisy breathing. PMHx of 3 episodes of PNA. When pt has a URI, mom states that it seems to take her a long time to get over it.

Croup CXR = steeple sign dexamethasone racemic epi hospitalize and obs

2 y/o M presents for barking cough and inspiratory stridor. Sxs improved when brought outside into cool air. No PMHx.

Pterygium Sx and artificial tear drops

22 y/o surfer bro presents for the following. Has started impacting his vision.

Nasal polyps which completes the Sampters Triad (nasal polyps, asthma, salyiclic acid sensitivity). Must test patient for cystic fibrosis.

22 yo male presents with recent diagnosis of allergic rhinitis. Patient reports new onset hyposmia and hyponasal voice. Patient states that he has a known allergy to aspirin and history of asthma. What is the patients diagnosis and what does it mean with the combination of his history?

Ludwig's Angina due to Klebsiella which is common pathogen in DM patients. Initial treatment is airway management then IV amoxicillin.

24 yo F with history of Type 1 DM presents to clinic with a stiff neck and mouth pain. Her exam shows b/l tenderness to the submandibular space with edema with no lymphadenopathy. What is her likely diagnosis, causative pathogen, and treatment?

Dental caries

25 y/o F presents for an annual physical. No PMHx but pt reports eating lots of sweets and drinking mostly sodas.

Acute dacrocystitis Treatment is oral clindamycin

28 yo M presents with fever and pain to the L inner eye region that began 3 days ago. Patients exam shows edema and erythema to the L medial canthal region. What is his likely diagnosis and treatment?

Ocular foreign body Fluorescein/ slit lamp + rust ring Topical abx if under eyelid Referral ASAP + broad spectrum abx for corneal FB

29 y/o M presents for the following. Stwates that while mowing the lawn, felt like something flew in his eye.

AOM Self limiting, Amoxicillin if bacterial

3 y/o presents for R ear pain and fever. No PMHx. PE + diffuse TM erythema and bulging

Peritonsillar Abscess An emergent consult due to need for possible I&D Treat with IV/oral amoxicillin or Clinda depending on severity

32 yo M presents to ER with severe right sore throat. She was diagnosed with cellulitis recently but states that the fever never resolved. During exam she is speaking with a "hot potato" voice and has severely swollen tonsils with uvular deviation. What is his likely diagnosis and treatment?

Epiglottitis Lateral XR w thumbprint sign ENT consult IV abx corticosteroids

4 y/o M presents for rapid onset of sore throat and fever. PE + stridor and difficulty speaking. Pt is ill appearing and leaning forward while sitting.

Chronic OM Removal of infected debris Ear plugs Topical abx drops Sx

4 y/o presents for L ear pain and fever. Mom has noticed clear serous fluid draining from the ear. PMHx of 3 episodes of AOM.

Septal deviation Nasal steroids +/- septoplasty

4 y/o presents for WCC. Mom reports that he has dryness and crusting around his nose regularly. PE + for turbinate hypertrophy.

Pseudomembranous Oral candidiasis which is a early manifestation of HIV. If all HIV testing and KOH prep is negative then treat with Nystatin swish and swallow and clotrimazole.

43 yo F presents with cotton like feel in her mouth for the past 3 days with ageusia. Physical exam shows white plaques that can be scrapped off to the buccal mucosa. What testing would you do to diagnose and treatment if ALL testing is negative?

Bacterial rhinosinusitis due to either strep or H. influenzae Treat with 10 days of either doxy or clinda + cephalexin due to penicillin allergy.

45yo M with known allergy to Amoxicillin presents with 1.5 weeks of clear nasal discharge, facial tenderness, and halitosis. Patient reports that his sxs are intermittent but worsened over past 3 days. What is the likely diagnosis, pathogen, and treatment?

Oral lichen planus There is no cure so treat with corticosteroids for pain management.

57 yo M presents with "roughness" to the mouth that began 1 week ago with white lesions to her b/l buccal membrane. Patient reports he has not been able to eat spicy food due to lesions. What is the most likely diagnosis and treatment?

Cholesteatoma (collection of skin in the middle ear) Sx + referral to ENT

6 y/o M presents for WCC. Mom reports that he has been well, though she does report that he has a foul smelling, brown d/c from his R ear. Denies pain or trauma. PMHx of recurrent OM and tympanovstomy tube placement.

Cerumen impaction Detergent ear drops (vinegar and ater) Mechanical removal or suction

65 y/o M presents for pressure and decreased hearing in his L ear. PE + significant accumulation of cerumen.

Retinal vein occlusion Referral VEGFi intravitreal implant retinal laser photocoag

66 y/o M w PMHx of HTN presents for sudden UL vision loss. Denies pain. PE + for retinal hemorrhages, cotton wool spots and blood and thunder appearance.

acute angle closure glaucoma IV diuretic referral ASAP

66 y/o farsighted M presents for UL vision loss and halos around lights. He reports rapid onset of extreme pain after leaving the movie theater. PE: +high IOP + non reactive pupil + cloudy cornea + dilated BV in the eye

Leukoplakia because hx of dentures and EtOH use but is considered precancerous. MUST biopsy to differentiate from erythroplakia. Refer to ENT as ithere are no approved therapies.

67 yo M presents with areas of white plaques to the oral mucosa that are 3cm in size upon exam. Patient reports his of denture use and is a recovering alcoholic. What is his likely diagnosis and a major concern?

COAG Prostaglandin analog eye drops Laser trabeculoplasty

68 y/o AA F w PMHx of DM presents for BL vision loss, progressing over the past 2 years. Denies pain. PE + for cupping of optic discs and increased IOP.

Wet (exudative) macular degeneration VEGFi intraocular injections long term photodynamic therapy

68 y/o F presents for gradual loss of vision in the central aspect of her vision over the past year. PE + neovascularization.

Polypoid Corditis (Reinke's Edema) Sx D/c smoking

68 y/o F w PMHx of tobacco dependence presents for voice changes. Referred to ENT with the following findings.

Malignant OE Tx DM, Tx for 6 mo

69 y/o F w PMHx of DM and breast cx in remission presents for intense R ear pain and d/c. Pain is worse at night and when eating. Pain radiates to the TMJ.

Papillomatosis Excision

7 y/o presents for progressive hoarseness over the past month. PE + stridor and the following.

Carcinoma of the larynx ENT for laryngoscopy + bx CXR and CT Endoscopic sx, radiation, laryngectomy

71 y/o M w PMHx of tobacco and ETOH dependence presents for new hoarseness of voice, ear and throat pain and wt loss. PE + for neck mass. No PMHx.

Dry atrophic macular degeneration Zinc, antioxidant vitamins (no specific tx)

75 y/o F presents for BL gradual and painless, central vision loss. States that images are wavy and distorted. PE + yellow retinal deposits.

Mastoiditis Admit for IV abx Surgical drainage if other tx fails

8 y/o F presents for pain and redness behind her R ear. Is feverish in clinic. No PMHx, but mom says she has had two episodes of R ear pain over the past year, not treated.

Retinal detachment Ophthalmoscope + decreased IOP Referral ASAP + Sx (urgent if macular is part of retinal detachment)

86 y/o pt presents for sudden UL vision loss. States that she started losing vision at the bottom of her eye, and vision loss gradually rose up her eye. Before the event, she reports seeing floaters. PMHx of osteoporosis and 10 days s/p cataract sx.

Mumps Symptomatic tx + isolation

9 y/o presents for UL facial swelling and lock jaw. Sxs were proceeded by fever, fatigue and HA. Mom is an anti-vaxxer.

Chronic gingivitis as a result of chronic plaque buildup can initially lead to A Periodontitis B Dental caries C Glossitis D Oropharyngeal cancer E Oropharyngeal candidiasis

A Periodontitis

A 47-year-old male presents to the clinic complaining of severe pain in his lower jaw, fever, and facial swelling. He describes a constant, throbbing pain which prevents him from eating or sleeping because the pain intensifies if he tries to lie down. The patient's breath exhibits a foul odor and there is an erythematous, fluctuant, tender mass near the left lower third molar. What is the most likely diagnosis? A dental abscess B halitosis C mumps infection D sialadenitis

A dental abscess

Your patient is a 54 year old male that presents with a runny nose and cough. The patent is also complaining of vertigo, tinnitus, and right sided hearing loss. Which of the following is the most likely diagnosis? a. labyrinthitis b. acute vestibular neuritis c. cerumen impaction d. BPPV e. Meniere disease

A - Labyrinthitis - Both vestibular neuritis and labyrinthitis have vertigo with usually a viral syndrome proceeding it. The key characteristic is there is no hearing loss associated with acute vestibular neuritis. Note: not cerumen impaction bc it will not usually cause tinnitus and usually doesn't have a viral syndrome associated w/ it. Not BPPV bc there are recurrent episodes of vertigo that are associated with predictable head movements. Not Meniere's bc Meniere's has longer duration of vertigo

Your patient is a 68 year-old female that presents with several months of progressively worsening central vision loss. Physical exam reveals Drusen Spots of the eye. Which of the following is the most likely diagnosis? a. macular degeneration b. open angle glaucoma c. optic neuritis d. cataracts e. uveitis

A - Macular degeneration: has drusen spots, central vision loss, gradual onset, occurs more in older ages - Not optic neuritis: pain is present w. optic neuritis - Not cataracts: no drusen spots - Not open angle glaucoma: peripheral vision loss present w/ this - Not uveitis: has pain, no Drusen spots, more common in ages 20-50 yos

Your patient is a 54 year-old male that presents with headaches, decreased hearing out of the right ear, right side facial paresthesias. The MRI of brain reveals a 2 x 3 cm right sided acoustic neuroma. Which of the following is not true regarding acoustic neuromas? a. Chemotherapy is the cornerstone of therapy b. Bilateral acoustic neuromas should increase a suspicion for neurofibromatosis c. Larger tumors may cause ataxia d. You can have decreased taste or taste disturbances with these e. Can cause constant vertigo

A - Treatment management options for acoustic neuroma include surgery, radiation, and observation. Larger tumors can cause compression of adjacent structures which lead to facial and trigeminal nerve pain, and paresthesias. Because these nerves innervate the tongue they can affect taste. Acoustic neuromas are known to cause constant vertigo

A 39-year-old woman comes to your office with a 2-day history of "unrelenting dizziness." The room feels like it is spinning, and there is associated nausea and vomiting. There has been no hearing loss, tinnitus, or sensation of aural fullness. The patient has just recovered from an upper respiratory tract infection. The patient is afebrile. On examination, nystagmus is present. The slow phase of the nystagmus is toward the left, and the rapid phase of the nystagmus is toward the right. There is a significant ataxia present. A. Vestibular neuritis B. Acute labyrinthitis C. BPPV D. Orthostatic hypotension

A - Vestibular neuritis: no hearing loss; acute, rapid onset (2 days); nausea, vomiting, recovered from URI, nystagmus present in PE; ataxia (gait instability) - Not acute labyrinthitis: hearing loss would be present with this disease - Not BPPV: movement is not the trigger, need shorter durations of dizziness; VN is an ongoing dizziness - Not orthostatic hypotension: doesn't cause nausea, vomiting, and "unrelenting dizziness"

Your patient is a 23 year old female that presents with white plaques on tongue. She has no history of chemotherapy, dentures, oral inhaled glucocorticoids, or radiation therapy. She is not a diabetic. Which of the following is the most likely etiology of her symptoms? a. HIV b. smokeless tobacco c. aphthous ulcers d. HSV e. sialadenitis

A - Whenever there is unexplained thrush (oral candidiasis) is a healthy individual, HIV should be considered as an etiology. Aphthous ulcers = produce ulcers not plaques HSV = presents as vesicles smokeless tobacco = cause of oral leukoplakia (has painless lesions) Sialadenitis = no symptoms on tongue

Nasal septal perforation

A 15 yo F with recent history of nasal surgery and afrin use presents with dry nose and crust. What is her diagnosis?

Malignant otitis externa can caused osteomyelitis Treatment is aggressive with IV/oral cipro

A 19yo F presents with painful aural discharge that began 3 days ago. She reports otalgia and her exam shows friable tissue within the canal. What is your diff dx, major concern and treatment plan?

MRI to evaluate for Thyroglossal duct cyst

A 23 yo M presents with an area of swelling to his neck. His exam shows a midline cyst to the anterior neck. What do you do next to diagnose and what is you differential dx?

Allergic conjunctivitis Add cool compresses and artificial tears, advice pt to avoid known allergens

A 23 yo M with hx of allergic rhinitis presents with itching to the b/l eyes and rhinorrhea that began 3 days ago. Patient reports he has been using otc oral antihistamines to minimal relief. What is his likely diagnosis and was tx would you add to pre-existing regime?

Vestibular neuronitis/Labyrinthitis Vestibular neuritis or labyrinthitis presents with vertigo, nausea, and vomiting, but not hearing loss or tinnitus. It is related to viral URIs, and develops over several hours, with symptoms worse in the first day, with gradual recovery over several days.

A 23-year-old graduate student presents with sudden onset of severe dizziness, with nausea and vomiting for the past couple of hours. She denies hearing loss or tinnitus. She has had a recent cold.

You have concern for Neural hearing loss and can perform a Rinne to determine if hearing loss is conductive or sensorineural. Official hearing loss must be determined via audiometric testing.

A 27 yo F with a PMHx of multiple sclerosis presents with hearing loss that began a few weeks ago. She states that it has progressed and has concern for permanent loss. What is your first diagnostic test and diff dx?

Peripheral Veritgo Fatiguable horizontal nystagmus

A 29 year old M presents to the clinic with sudden onset "dizziness". He is also complaining of tinnitus and slight hearing loss. What do you expect to find upon performing a Dix hallpike and your diff dx?

Facial numbness or Frey's Syndrome (sweating instead of salivating)

A 29yo F presents to the office for a salivary gland tumor removal. When explaining the procedure what is a common complication that you mention?

Brainstem auditory evoked response testing is best as the child might be unable to sit for a formal audiometric test. If pts hearing loss is significant then you consider a cochlear implant long term as it MAY be a congenital defect.

A 3 yo M presents with his mother to the clinic. Mom states that the patient doesn't seem to be able to understand was she is saying and she has concern for hearing loss, states that her sister is deaf. What testing do you consider and possible treatment?

Allergic Rhinitis Add Beclomethasone corticosteroid nasal spray bc it is the most effective maintenance therapy. Tell pt to remove allergens from home.

A 30 yo female patient presents with sneezing, and rhinorrhea that has been ongoing for the past 2 weeks. Patient reports a history of similar sxs and that she has a FHx of atopy. Patient has been take oral loratadine but states that her symptoms are not improving. What is the diagnosis and what treatment would you add to her current regime?

B Diuretics and low-sodium diet Diuretics and a low sodium diet will decreases the endolymphatic pressure in the semicircular canals, which is believed to be elevated in Meniere's disease, and help relieve symptoms.

A 34 year-old female presents with recurrent bouts of dizziness, tinnitus, and hearing loss. She states that the episodes are incapacitating and cause her to become nauseous and vomit. The attacks last about one hour and the symptoms disappear after a few days. The last two episodes were treated with meclizine (Antivert) and prochlorperazine (Compazine) at the emergency room. Audiologic testing reveals low-tone frequency hearing loss. Which of the following is the most appropriate long-term management for this patient? A Epley maneuver B Diuretics and low-sodium diet C Broad-spectrum antibiotics and Ibuprofen D Scopolamine transdermal patch

infectious keratitis treatment is dependent on gram stain but likely a fluoroquinolone based on her contact use.

A 34 yo F presents with purulent discharge to the RIGHT eye with round white spots to the cornea. Patient reports she will tend to sleep in her contacts frequently. Her exam lacks hypopyon but you note her improper contact use. What is her likely diagnosis and treatment?

Bactieral Sialadentis, likely pathogen is S. Aureus Treatment is to increase salivary flow with stimulating lozenges, massages, warm compress Oral abx for mild infection IV Naficillin if SEVERE infection

A 34 yo M presents to the clinic with 4 days of unilateral swelling to his RIGHT jaw region. He states that pain worsens with eating. His exam shows palpable tenderness to the opening of a major duct to the mouth where purulent drainage was aspirated. What is a likely diagnosis and treatment?

Oral lichen planus, erythoplakia, leukoplakia

A 34 yo M presents to the clinic with plaques to his buccal membrane for the past 2 weeks. Patient denies burning his mouth in anyway but the plaques will change color randomly. You tell the patient that you must biopsy them. What are you differential diagnoses?

Orbital cellulitis urgent IV naficillin and clindamycin Major concern is permanent vision loss

A 34yo M presents to the ER with some vision loss, eyelid swelling and ocular pain. Upon taking his hx he mentions that he recently completed treatment for a maxillary sinusitis but was still feeling sick. He was febrile during exam which shows slight proptosis. What is a major concern, treatment plan, and diff dx?

Menieres disease Treatment is meclizine and low salt diet/diuretics to drain the fluid in the inner ear Diagnostic testing is audiogram and caloric testing

A 38 yo F presents with episodic tinnitus, hearing loss, and vertigo. She reports that sxs do not worsen with sudden movement. What is her likely dx and tx?

Dry age related macular degeneration treatment is vitamin C, D, zinc, and copper as well as vascular endothelial growth factor inhibitors

A 43 yo F presents with gradual b/l vision loss and ongoing difficulty seeing when driving at night. Her exam shows drusen with retina atrophy. What is her likely diagnosis and treatment?

Posterior Blepharitis Treatment is mechanical was warm compresses and Meibomian gland massage Can recommend artificial tears

A 43 yo F presents with greasy and oily scales to her eyelids that began 3 days ago. She reports that the lid turned inwards today which was very uncomfortable. What is her likely diagnosis and treatment?

Migraine associated dizziness

A 45 yo patient presents with tinnitus and vertigo. She denies hearing loss of any kind and states that the vertigo is episodic. What do you suspect as a diff dx?

Laynygitis and treatment is vocal rest with speech therapy Can use glucocorticoids to speed up treatment but there is a risk of vocal damage.

A 45 yoM presents with breathiness and a vocal tremor that began 1.5 weeks ago. He states that his sxs only worsen when he speaks but notes associated pain. What is a likely diagnosis and treatment plan?

Acoustic neuroma

A 52-year-old male presents with unilateral left-sided hearing loss that has progressed over 2 months. His wife states that he has difficulty discriminating words, although he can hear the sounds. He has also experienced some imbalance lately, but he contributes this to trying to do too many things too quickly.

Chalazion but might want to biopsy to r/o BCC due to ongoing sxs. Treatment is warm compresses if needed but typically will self-resolve

A 56 yo M presents with a firm, small nodule to the lower eyelid that has been present for the past few months. Exam shows mild erythema without warmth or fluctuance. What is his likely diagnosis, your first course of action, and major concern due to ongoing sxs?

The patient likely have Sudden Sensorineural Hearing loss with is the only auditory emergency and idiopathic. You have concern for a clot that caused the sudden painless hearing loss.

A 56 yo M presents with sudden, painless, unilateral hearing loss. You automatically think to treat the patient with oral steroids, vacyclovir, and perform a advanced imaging why?

Sensorineural hearing loss, consider hearing aids

A 65 yo F presents to the clinic with slight hearing loss. Upon her Rinne exam, she states that she hears the sound in front of her ear better than behind. What is her likely diagnosis and treatment?

Due to age and audiometric test, sensorineural hearing loss if most likely. Treatment is hearing amplification with hearing aids.

A 78 yo M presents with slight hearing loss that began a few months ago. Patient denies any ear pain, ringing, or vertigo but states that is has gotten progressively worse. A formal audiometric test shows diminished threshold between the bone and air readings. What is his likely diagnosis and treatment?

What is the difference between a chalazion and hordeolum?

A chalazion is painless and involves Meibomian or Zeis obstruction, while a hordeolum is painful and involves follicle/moll/zeis if external and meibomian if internal.

What is the treatment for a chalazion vs Hordeolum

A chalazion requires conservative/hygiene and a hordeolum needs a warm compress/ I&D in 48 hours if it doesn't drain. Active drain needs erythromycin or bacitracin topically.

Conductive hearing loss

A patient presents complaining of gradual hearing loss over the past 3 months in the right ear. He admits to the use of Q-tips and otherwise does not wear earplugs or place other foreign objects in his ear. Weber test lateralized to the right and Rinne showed better bone conduction on the right.

Must wait until 3 months after the injury in order to perform an open reduction.

A patient presents to clinic 3 weeks post nasal trauma which shows fracture on xray. What is the best treatment?

A Benign positional vertigo Benign positional vertigo is characterized by the sudden onset of vertigo when rolling onto the affected side or tilting the head up. The typical duration is less than a minute. There can be associated nausea and vomiting. There is no hearing impairment or tinnitus.

A patient presents with complaint of sudden onset of recurrent episodic vertigo for one week that happens when rolling onto the left side. The patient states that this sensation lasts approximately 30 seconds and then goes away. The patient admits to associated nausea. The patient denies associated hearing difficulties or tinnitus. Which of the following is the most likely diagnosis? A Benign positional vertigo B Ménière's disease C Acoustic neuroma D Vestibular neuronitis

NO auditory sxs but gradual onset vertigo Possibly non-fatiguable vertical nystagmus upon Dix hallpike maneuver.

A patient was referred to you due to concern for Central vertigo. What do you except the patients sxs to be?

A 4 year-old child presents with a rapid onset of high fever and extremely sore throat. Which of the following findings are suggestive of the diagnosis of epiglottitis? A Croupy cough and drooling B Thick gray, adherent exudate C Beefy red uvula, palatal petechiae, white exudate D Inflammation and medial protrusion of one tonsil

A. Croupy cough and drooling

Which of the following is a staphylococcal infection characterized by a localized red swollen and acutely tender abscess of the upper or lower eyelid? A. Hordeolum B. Uveitis C. Chalazion D. Dacryocystitis

A. Hordeolum

A 32 year-old female presents complaining of spiking fevers. She was seen four weeks ago with a complaint of left ear pain and was treated for otitis media. She continues to have symptoms, but now has pain behind the ear. On examination you note left post auricular tenderness and erythema. Which of the following is the treatment of choice in this patient? A. IV antibiotics B. Mastoidectomy C. IM steroids D. Ventilating tube placement

A. IV antibiotics

A 58 year-old patient presents with spells of dizziness which is described as a spinning sensation. This has occurred several times a day for the last month. The patient also complains of some mild hearing loss, fullness, and a blowing sound in the right ear. Which of the following is the most likely diagnosis? A Meniere's syndrome B Labyrinthitis C Benign paroxysmal positioning vertigo D Vestibular neuronitis

A. Meniere's syndrome

A 14 year-old male presents with complaint of worsening sore throat for two weeks. He now complains of fever, difficulty swallowing, and difficulty opening his mouth. The patient's mother states his voice seems muffled. On examination his left tonsil is bulging and the uvula is displaced to the right. Which of the following is the most appropriate management? A Needle aspiration B Corticosteroid administration C Nebulized epinephrine administration D Nasotracheal intubation

A. Needle aspiration

An otherwise healthy college student presents with complaints of cough, malaise, fever, sore throat, yellow-green nasal discharge, and headache for the past 3 weeks. She complains of facial pain over the cheeks and while chewing. On examination, the nasal turbinates are erythematous and edematous nearly obstructing the nares bilaterally. Her face is tender to palpation over the maxillary sinuses. She has not been previously evaluated. Which of the following is the next best step in the management of this patient? A Prescribe amoxicillin/clavulanate (Augmentin) B Prescribe azithromycin C Prescribe trimethoprim-sulfamethoxazole (TMP-SMX) D CT Head E Sinus aspiration

A. Prescribe amoxicillin/clavulanate (Augmentin)

A 24-year-old mixed martial arts practitioner presents after an injury during a match which has resulted in a fluctuant, mildly tender edematous lesion of the anterior-superior outer portion of the right pinna. Which of the following is the most appropriate treatment? A. Refer to otolaryngology for definitive treatment with I & D and pressure dressing B. Perform I & D only if the pinna becomes erythematous and extremely tender C. Prescribe a 10-day course of amoxicillin/clavulanate (augmentin) and schedule a follow-up appointment in 2 weeks D. Apply a soft bulky dressing to the pinna and recommend no further treatment, but have the patient follow-up only if he develops a temperature greater than 101 degrees

A. Refer to otolaryngology for definitive treatment with I & D and pressure dressing

An 85-year-old nursing home patient was seen in a local physician's office during the day for a corneal abrasion. The patient had antibiotic drops instilled, and the eye was patched. At 10: 00 p.m., the nursing staff calls reporting the patient is very confused. The most appropriate action is to A. Remove the eye patch B. Prescribe haloperidol C. Have the patient taken to the emergency room D. Reassure the nursing staff and see the patient the next day

A. Remove the eye patch

You are called into the ER to see a patient that was hit by a motorcycle while crossing the street. Examination of her right eye reveals eyelid swelling, decreased visual acuity, enophthalmos (sunken eye). She denies headache or loss of consciousness but has significant pain in her eye and cheek. You order a CT of her head, what finding do you suspect? A. a traumatic deformity of the orbital floor B. Bleeding in the space between the brain and the tissue covering the brain. C. a collection of blood inside the front part of the eye D. Corneal abrasion

A. a traumatic deformity of the orbital floor

A 17-year-old girl comes to your office with a 1-day history of red eye. She describes not being able to open her right eye in the morning because of crusting and discharge. The right eye feels swollen and uncomfortable, although there is no pain. On examination, she has a significant redness and injection of the right bulbar and palpebral conjunctivae. There is a mucopurulent discharge present. No other abnormalities are present on physical examination. Her visual acuity is normal. A. bacterial conjunctivitis B. viral conjunctivitis C. allergic conjunctivitis D. autoimmune conjunctivitis

A. bacterial conjunctivitis

A 23 year-old sexually active female presents with a 4 day history of painless bilateral eye exudates which she describes as copious. Visual acuity is 20/20, generalized conjunctival inflammation with sparing of the cornea is noted on physical examination. Gram stain of the exudate reveals gram negative diplococci. Appropriate management of this case is A. ceftriaxone (Rocephin) B. polymyxin ophthalmic drops (Aerosporin) C. ciprofloxacin (Cipro) D. doxycycline (Doryx)

A. ceftriaxone (Rocephin)

A patient with Type 1 diabetes mellitus was treated for otitis externa of the right ear for 2 weeks with topical ear drops. The patient presents today with persistent, foul aural discharge, granulations in the ear canal, and deep ear pain. Which of the following is the proper treatment at this time? A. ciprofloxacin (Cipro) IV B. cefuroxime (Zinacef) IV C. ampicillin-sulbactam (Unasyn) PO D. azithromycin (Zithromax) PO

A. ciprofloxacin (Cipro) IV

A 12 year old present to your office with red eyes, itching and tearing bilaterally. He has a past medical history significant for asthma. As you examine the inner eyelid what finding do you expect to see? A. cobblestone mucosa B. Kayser-Fleischer rings C. mucopurulent discharge D. dendritic ulcerations

A. cobblestone mucosa

An 45 year old man comes in complaining of hearing loss in his left ear. He says his ear is ringing, everything is "spinning," he has a HA and his face feels numb. Weber testing lateralizes to his right ear and AC>BC on Rinne testing. What is it? What tests to order to be sure? What is the treatment?

Acoustic neuroma MRI = TOC, audiometry surgery or focused radiation

a 60-year-old Asian American woman presents with sudden ocular pain. She reports she was visiting the planetarium when the pain started and when she walked outside she saw halos around the street lights. The pain was so bad that she began to vomit. She reports her vision is decreased. Physical examination reveals conjunctival injection, a cloudy cornea, and pupils What is the dx?

Acute Angle glaucoma

a 3-year-old previously healthy male is brought to your office by her mother. The mother reports the child has been crying and pulling at her right ear over the past 2 days and reports the patient has been febrile the past 24 hours. The patient's past medical history is unremarkable, although the mother reports the patient had a "common cold" a week ago which resolved without intervention. His temperature is 101.6 F, blood pressure is 100/70 mmHg, pulse is 120/min, and respirations are 22/min. The otoscopic exam is seen here. Dx: Tx:

Acute OM - Amoxicillin -Augmentin (on abx w/in 30 days) -Cefixime (children) -Azithromycin (PNC allergy)

An 18 month old boy is brought in by his aunt because he won't stop crying or tugging on his ear. His aunt is his main caregiver and you notice that she smells like cigarette smoke. The child was not breast fed. You find out that he just got over an upper respiratory infection. PE and Pneumatic otoscopy reveals decreased TM mobility, a bulging & erythematous TM w/ effusion and loss of landmarks. What is it? Main bugs? Tx?

Acute Otitis Media S. PNA MC H. influenzae M-cat Group A Strep amoxicillin due to <2

A 40 year old female comes into the office complaining of difficulty hearing. The patient states that she has had an URI for the past 4 days. On exam you note conjunctival inflammation, otalgia, fever, irritability and cervical lymphadenopathy. The tympanic membrane is erythematous, with poor light reflex, decreased landmarks, and cloud, purulent fluid behind the TM. Dx: Pathogen: Tx:

Acute Otitis media -S. pneumoniae, H. Flu, S. Pyogenes, M. catarrhalis -Antipyretics -Abx (amoxicillin, augmentin) -PNC allergy: azithromycin -F/U in 3 days if cont pain/fever -Decongestants -Avoid antihistamines

A 14-year-old boy with no significant past medical history presents 3 days after developing a red, irritated right eye that spread to the left eye today. He has watery discharge from both eyes and they are stuck shut in the morning. He reports recent upper respiratory symptoms and that several children at his day camp recently had pink eye. He denies significant pain or light sensitivity and does not wear contact lenses. On examination, his pupils are equal and reactive and he has a right-sided, tender preauricular lymph node. Penlight examination does not reveal any corneal opacity.

Acute conjunctivitis

A 6-year-old girl with no significant past medical history presents 4 days after developing a red, irritated left eye. Her mother states that she has been wiping thick whitish-yellow discharge from her eye, and the eye is matted shut in the morning. She denies any exposure to a sick contact, upper respiratory symptoms, or contact lens use. She also denies any significant pain or light sensitivity. On examination, the patient's pupils are equal and reactive. She does not have a tender preauricular lymph node. Penlight examination does not reveal any corneal opacity, but thick, whitish discharge is seen.

Acute conjunctivitis

A 68 year old female comes in with facial pain that is "really bad when I pick things up and I have a bad headache. This is making me feel tired." She works as a cleaner at a hotel. This has been a problem for 15 days PE reveals a fever and purulent nasal d/c. She thought it was getting better, but then things seemed to get worse. She was sick a little over a 2 weeks ago. Hx reveals that she smokes a pack a day. What is it? What are the bugs? What is the tx?

Acute rhinosinusitis S. PNA H-flu M-cat, & group A Strep augmentin b/c > 10-14 days

An 8 year old girl was brought in by her mother b/c her "eye hurts when she looks around." She is experiencing some double vision and her eyelid looks swollen & red/eye looks bulging. Hx reveals she just had a sinus infection A CT confirms orbital cellulitis, as you suspected clinically, which is an emergency - what are the next steps?

Admit the patient Start an IV and give vanco add on one of: ceftriaxone or cefotaxmine can also do: unasyn, zosyn or clindamycin

A 75-year-old woman presents with new-onset distortion in one eye. Vision is 20/80 in the involved eye. She has a family history of AMD and has smoked 20 cigarettes a day for most of her adult life.

Age-related macular degeneration

a 13-year-old boy with clear fluid discharge from his nose for 2 days duration. This has also been associated with sneezing. On nasal exam, the mucosa and turbinates appear edematous and slightly bluish, he has swollen dark circles under his eyes, and a transverse nasal crease. Dx: Tx:

Allergic Rhinitis - IN corticosteroids (fluticansone) -Oral H1 recp. antagoinsit (fexodenadine or azelastine) -Allergen Free environment -ENT referral if drug failure

A 32 year old man comes in the clinic because he feels "very congested this time of the year and wants to do something about it this time." He says he sneezes a lot and and says his nose drips "clear stuff." PE reveals cobblestone mucosa of conjunctiva, purple around his eyes and a nasal bridge crease. You do not see any polyps. What is it? What is first line treatment?

Allergic Rhinitis Intranasal corticosteroids

A woman comes into the clinic with SUPER itchy red eyes.... She has a Hx of asthma and eczema. Her vision is normal. On physical exam, you notice cobblestoning on the inner/upper eyelids. What is it and how do you treat it?

Allergic conjunctivitis Olpatadine Pheniramine-Naphazoline Emedastine.

Patient will present as → a 13-year-old boy with discharge from his nose for 2 days duration. This has also been associated with sneezing. On nasal exam, the mucosa and turbinates appear edematous and slightly bluish pale, he has swollen dark circles under his eyes, and a transverse nasal crease (nasal salute).

Allergic rhinitis

A 60 year old male comes into the office with complaints of R eye vision loss. He describes it as if a curtain descended for a few moments, then his vision in his R eye came back. What is the appropriate dx?

Amaurosis Fugax

Patient will present as → an 82-year-old man presents to the emergency department complaining of vision loss in his left eye. He states that it suddenly appeared as if a curtain was coming down over his left eye. It resolved after five minutes, and his vision has returned to normal. He has a history of coronary artery disease and type 2 diabetes. What is the dx?

Amaurosis fugax

a 5-year-old male is brought by his parents and referred by his teacher for suspected decreased vision in his left eye. His mother had not noticed any vision problems. He has had normal growth and development. On exam, the patient has an abnormal vision screen of the left eye and red reflex asymmetry. DX:

Amblyopia

A 3 year old girl is brought in by her parents because an eye test at day care revealed that she has issues seeing out of one eye. Hx reveals she was born prematurely. It seems that this issue may have been a problem for a while. Eye testing shows >/- 2 line difference in visual acuity b/w 2 eyes. What is it and what is the treatment?

Amblyopia 1 correct refractive error 2 occlude good eye to strengthen other 3 penalization w/ dilation drops twice weekly Cyclogel or atropine 4. vision therapy - train oculomotor skills

A 65 y/o male comes into the office today with gradual central vision loss. On exam you note drusen spots. What test would you run next, and what would you expect to see?

An amsler grid; metamorphsia

A 50-year-old woman, who has no eye symptoms, is found during routine ophthalmic examination to have elevated intraocular pressure of 42 mmHg in both eyes. Funduscopy shows that the optic nerve head appears normal, with no evidence of glaucomatous neuropathy. Gonioscopy shows that the anterior chamber angles are closed for almost the full circumference.

Angle-closure glaucoma

A 64-year-old woman presents to the ER with severe pain around her right eye of 4-hour duration, accompanied by blurred vision in that eye. She is also nauseated. Examination shows a red right eye with edematous cornea and a wide pupil that is unresponsive to light. Intraocular pressure is extremely elevated (60 mmHg), only in the right eye. The anterior chamber angle is closed in both eyes.

Angle-closure glaucoma

What is the most common type of epistaxis?

Anterior from Kiesselbach's plexus

How would you treat a patient with nasal polyps?

Antihistamines Topical nasal corticosteroid

A 17 year old female patient comes in after a dentist visit, concerned about small mouth ulcers that "hurt a little." She has no PMH and has never been sexually active. PE reveals shallow, roundish ulcers with a red halo on the buccal mucosa of her right cheek. What is it? What is first line Tx?

Aphthous Ulcer (Canker sore/ulcerative stomatitis) glococorticoids: Clobetasol gel or oint) Dexamethasone elixir swish & spit, Triamcinolone in orabase

A 23-month-old male presents to the office with his father who reports the patient has had an acute onset of severe pharyngitis, fever of 103.5 degrees F and what sounds like harsh, high-pitched breath sounds. His dad states the child has started drooling and seems to be worsening. The child is not presently crying but has muffled voice sounds. The child has not been immunized due to religious reasons. What is the most appropriate next step? A Administer ceftriaxone (Rocephin) B Emergent transfer C Administer racemic epinephrine D Obtain intravenous access

B Emergent transfer

A 26-year-old female presents with a whitish coating on her tongue and lips. When you attempt to rub the plaque with a tongue depressor, a small amount of bleeding is noted from the oral mucosa. Her past medical history includes asthma. What diagnostic test would you perform to confirm your diagnosis? a. a culture and sensitivity b. a wet mount potassium hydroxide preparation c. a gram stain d. a rapid strep test e. Tzanck smear

B (oral thrush/oral cnadidiasis) a wet mount potassium hydroxide preparation will reveal fungal spores and nonseptated mycelia

Your patient is an 18 year-old female that presents with right side purulent otorrhea. Visualized tympanic membrane on the right side reveals a small perforation. Which of the following is the most appropriate treatment regimen? a. Floxin otic drops b. Amoxicillin and Floxin otic drops c. Zithromax d. Amoxicillin

B - Amoxicillin & Floxin otic drops: (CSOM) helps with a bad infection; when there is a perforation the ear canal is contaminated w/ the purulent drainage; just floxin otic drops or amoxicillin is not powerful enough; Zithromax is not powerful enough either

Your patient is a 3 year-old male that presents with a bulging right tympanic membrane and a fever of 103.0. He has just finished a course of amoxicillin 14 days ago. Which of the following is the most appropriate treatment? a. High-dose amoxicillin b. Augmentin c. Keflex d. Cipro

B - Augmentin (pt is at risk for beta-lactam resistance); Clavulanate is a beta-lactamase inhibitor Infection: Acute otitis media - Not A: already tried that - Not D: Cipro should be avoided in this age group (could cause joint pain) - Not C: This patient is at risk for beta-lactam resistance, therefore Keflex and amoxicillin should not be used. It is too closely related to Amoxicillin = beta-lactam

Which of the following is the most appropriate management for an auricular hematoma? a. observation b. incision and drainage c. ice for 20 min 4 times per day d. start cephalexin e. start cefuroxime

B - Auricular hematomas should be drained as soon as possible. Delay in treatment can lead to a cauliflower ear that is permanent deformity of the fibrocartilage of the ear that is an overgrowth.

Your patient is a 63 year old female that has a history of HTN and CAD. She has had a 3 month history of recurrent episodes of vertigo. These episodes tend to occur when she lays back in bed with her head not propped up. Her ENT exam is unremarkable. She has had an MRI of her brain that was negative as well as carotid dopplers. Which of the following is the most likely diagnosis? a. Meniere's disease b. BPPV c. Vestibular neuritis d. cerebellar infarct e. otosclerosis

B - Benign Paroxysmal Positional Vertigo tends to have brief recurrent episodes of vertigo with predictable head movements or positions. There is no auditory symptoms. BPPV usually has a positive Hallpike Maneuver. Meniere's disease usually has hearing loss and tinnitus with vertigo as well as longer duration of vertigo Cerebellar infarct = r/o bc negative MRI VN = acute, rapid onset of vertigo otosclerosis = PE is positive; BPPV = nothing significant

Your patient is a 23 year old female that presents with swelling and erythema to the upper eyelid on her right eye. Her extra ocular movements are intact. There is no pain with extra ocular movements. She also has a history of Rosacea. Which of the following is the most likely diagnosis? a. Hordeolum b. Blepharitis c. Chalazion d. Dacrocystitis e. Viral conjunctivitis

B - Blepharitis is characterized by swelling of the eyelid. Blepharitis is best treated with warm compresses and topical antibiotics such as bacitracin, erythromycin, or azithromycin. Chalazion is an inflammatory lesion that develops with Meibomian glands becomes obstructed. Hordeolum (stye) is a painful red lump on the eyelid when the gland on the edge of the eyelid gets inflamed. Dacrocystitis is caused by the nasolacriminal gland being blocked Viral conjunctivitis = inflammation of the conjunctiva caused by viral infection

Which of the following is not true about peritonsillar abscess? a. Group A Streptococcus is the most common organism b. MRI of the soft tissue neck is the imaging study of choice for confirmation c. Appropriate empiric abx therapy includes clindamycin or Unasyn d. Vancomycin is an appropriate add-on abx if there are airway issues

B - CT scan of soft tissue of the neck with IV contrast is the imaging study of choice. Group A Streptococcus is the most common organism. There are also so cases of staph. Appropriate empiric antibiotic therapy includes Clindamycin and Unasyn. Vancomycin is an appropriate add-on antibiotic when there are airway issues.

Your patient is a 34 year-old male who presents with erythema and swelling to the upper right medial nasolacrimal duct. He has no fever. His extraocular movements are intact. He has no ectropion. Which of the following is the most likely diagnosis? a. Periorbital cellulitis b. Dacryocystitis c. Orbital cellulitis d. Hordeolum e. Scleritis

B - Dacryocystitis is caused by the nasolacrimal duct being blocked. The most common organism is alpha hemolytic strep, staph epidermis, and staph aureus. Empiric therapy with clindamycin and doxycycline or Bactrim is helpful. This can lead to periorbital cellulitis. - Not Hordeolum: on the eyelids (painful) - Not orbital cellulitis: pain present w/EOM w/ this disease - Not periorbital cellulitis: usually with URI's (no fever present here) - Not scleritis: common w/ autoimmune disorders, diffuse

Which of the following is the most common cause for hearing loss? a. Acoustic Neuroma b. Presbyacusis c. cerumen impaction d. vestibular neuritis e. labyrinthitis

B - Presbyacusis (most common cause of SNHL)

Your patient is a 69 year old female that presents with a sudden onset of floaters in the right eye, flashes of light, and floaters resemble a cobweb. There is no pain. Which of the following is the most likely diagnosis? a. optic neuritis b. retinal detachment c. pseudotumor cerebri d. macular degeneration

B - Retinal detachment: sudden onset of floaters, unilateral, flashes of light, age > 40, painless - pseudotumor cerebri = affects both eyes and will have HA's - macular degeneration: slower onset; not these visual field changes - optic neuritis: has more of a central blind spot rather than floaters

The most common organism isolated in periorbital cellulitis in vaccinated children in the absence of trauma is A. H. influenzae type B B. Streptococcus pneumoniae C. Moraxella catarrhalis D. Staphylococcus aureus E. Pseudomonas aeruginosa

B Streptococcus pneumoniae

Which of the following is the most common etiologic agent associated with acute bacterial sinusitis in the adult population? A Staphylococcus aureus B Streptococcus pneumoniae C Pseudomonas aeruginosa D Mycoplasma pneumoniae

B Streptococcus pneumoniae The typical pathogens associated with acute bacterial sinusitis are Streptococcus pneumoniae, other streptococci species, and Haemophilus influenzae. Moraxella catarrhalis and Staphylococcus aureus are less common causes. Pseudomonas is a less common cause and would be associated with nosocomial infection often in a critically ill patient. Mycoplasma is not a typical cause of bacterial sinusitis.

Which of the following conditions are not associated with allergic rhinitis? a. atopic dermatitis b. urticaria c. sinusitis d. asthma e. allergic conjunctivitis

B- urticaria; others are seen with allergic rhinitis

your patient is a 64-year-old male was found to have an opacity on his lens and decreased visual acuity. Which of the following is the most likely diagnosis? A Glaucoma B. Cataract C Astigmatism D Hyphema E Scleritis

B. A cataract is a defect in the lens. Cataracts are the leading cause of blindness in the world. The only treatment is surgical correction.

A 34 year-old female presents with recurrent bouts of dizziness, tinnitus, and hearing loss. She states that the episodes are incapacitating and cause her to become nauseous and vomit. The attacks last about one hour and the symptoms disappear after a few days. The last two episodes were treated with meclizine (Antivert) and prochlorperazine (Compazine) at the emergency room. Audiologic testing reveals low-tone frequency hearing loss. Which of the following is the most appropriate long-term management for this patient? A Epley maneuver B Diuretics and low-sodium diet C Broad-spectrum antibiotics and Ibuprofen D Scopolamine transdermal patch

B. Diuretics and low-sodium diet

A 32 year-old carpenter complains of right eye irritation all day after driving a metal stake into the ground with his hammer. He states that "something flew into my eye." Visual acuity is 20/20. Pupils are equal, round, reactive to light and accommodation. Extraocular movements are intact. There is minimal right corneal injection. No foreign body is noted with lid eversion. Fluorescein stain reveals a tiny pinpoint uptake in the area of the corneal injection. Which of the following is the most appropriate diagnostic test at this stage? A. MRI B. X-ray orbits C. Applanation tonometry D. Fluorescein angiography

B. X-ray orbits

A 26-year-old woman comes to your office with a 6-day history of severe dizziness associated with ataxia and right-sided hearing loss. She had an upper respiratory tract infection 1 week ago. At that time, her right ear felt plugged. On examination, there is fluid behind the right eardrum. There is horizontal nystagmus present, with the slow component to the right and the quick component to the left. Ataxia is present. What is the most likely diagnosis in this patient? & what is the treatment? A vestibular neuronitis B acute labyrinthitis C positional vertigo D orthostatic hypotension E Meniere disease

B. acute labyrinthitis; Rest & antiemetics if needed

A patient presents with a 3-day history of vertigo associated with turning over in bed, which lasts for several minutes. There are no other symptoms of the ear. Dix-Hallpike testing shows rotary nystagmus, which diminishes with repeated testing. Which of the following is the most likely diagnosis? A. central nervous system (CNS) lesion B. positional vertigo C. Labyrinthitis D. Meniere disease E. vestibular neuronitis

B. positional vertigo

DM Retinopathy symptoms

Background: micoranyrisms, hemorrhages, exudates Pre-proliferative : - Cotton wool spots, arteriolar ischemia Proliferative: -Neovascularization, macular edema

A 30 y.o comes in with R eye pain, reduced vision, photophobia, and perfused tears. On exam you note a central ulcer, the cornea is cloud, edematous eyelids and thick ropy mucopurulent discharge. DX TX

Bacterial Keratitis -Referral -Cycloplegic/ cold compress - Abx drops (mild/ mod) -Systemic abx (severe) (+) Cefazolin (-) Tobramycin or ofloxacin no organism cephalosporin + aminoglycoside (-mycine)

a 7-year-old boy is brought to his pediatrician for evaluation of a sore throat. The sore throat began 4 days ago and has progressively worsened. Associated symptoms include subjective fever, pain with swallowing, and fatigue. The patient denies cough or rhinorrhea. Vital signs are as follows: T 101.4 F, HR 88, BP 115/67, RR 14, and SpO2 99%. Physical examination is significant for purulent tonsillar exudate; no cervical lymphadenopathy is noted. Dx: Common Pathogen: Tx: Complications:

Bacterial Pharyngitis -Group A b-hemolytic strep -PO abx : PNC, cefuroxime Amoxicillin (child) -Hydration, rest, ibuprofen (pain/fever) -ENT referral if pt has recurrent infections -Rheumatic fever, ludwig's abscess

A patient comes in with eye pain, red eyes, a hazy cornea and c/o that he can't keep his eyes open. He reveals that he showers with his contacts in (eyes look red). he has a Hx of HIV. Slit lamp reveals increased fluorescent uptake. What is it what bug is suspect? What are the treatment steps? should you patch it?

Bacterial keratitis pseudomonas 1 get cx r/o SA/strep/pseudo 2 topical FQ 3 same day eye doc never patch it

Patient will present as → a 34-year-old previously healthy male with complaints of facial pressure and rhinorrhea for the past 3 weeks. The patient reports that several weeks prior, he had a "common cold" which resolved. However, he has since developed worsening facial pressure, especially over his cheeks and forehead. He reports over 1 week of green-tinged rhinorrhea. His temperature is 100.1 F (37.8 C), blood pressure is 120/70 mmHg, pulse is 85/min, and respirations are 15/min. The nasal exam reveals edematous turbinates and purulent discharge. The patient has facial tenderness with palpation over the involved sinus.

Bacterial sinusitis Why not viral? Green rhinorrhea indicates that the viral infection has clogged his sinuses and allowed it to turn into bacterial sinusitis. (ugh!)

An 18 year old male comes in after scuba diving all day b/c he has been experiencing pain, fullness and hearing loss that hasn't gone away after the dive. He had just gotten over a cold prior to the dive. He also mentioned that he felt blood came out of his ear. What is it? Tx?

Barotrauma autoinsufflation

A 65-year-old man presents with bilateral ocular itching, burning, and foreign body sensation, as well as crusting of his eyelids in the morning for the past several months. He also notes that his eyes are often red and feel dry. Physical exam demonstrates facial pustules, facial redness, and telangiectasia. Slit lamp exam shows lid collarettes, telangiectasia, and capped meibomian glands.

Blepharitis

A 20 year old male patient with Down Syndrome comes in with flaky eyelashes and says his eyelids "burn and are crusty." You see that his lids look scaly. What is it? what are the bugs? What is the tx?

Blepharitis SA/SE eyelid hygiene macrolides if bad

a 34-year-old female with crusting, scaling, red-rimming of eyelid and eyelash flaking along with dry eyes. The patient has a history of seborrhea and rosacea. Dx: Tx:

Blepharitis Tx: warm compress, baby shampoo, massage. Topical antibiotics (erythromycin, polymyxin/bacitracin)

a 13-year-old who was hit in the right eye by a baseball. The area is ecchymotic and swollen. He complains of pain, rated 6 out of 10. On physical exam, the patient has eyelid swelling, decreased visual acuity, enophthalmos (sunken eye), and anesthesia/paresthesia in the gums and upper lips DX TX

Blow out fx -Refer -CT scan -Nasal decongestins, Abx -Cold compress -Avoid sneezing -Surgical repair

Pt has had multiple vertigo episodes that last<1 min, does not have HL,& is triggered by certain movements, name dz? a. barotrauma b. vestibular neuritis c. BPPV d. ETD e. Meniere's

C

A 30-year-old man comes to your office for assessment of dizziness. The dizziness occurs when he rolls over from the lying position to either the left side or the right side. It also occurs when he is looking up. He describes a sensation of "the world spinning around" him. The episodes usually last 10 to 15 seconds. They have been occurring for the past 6 months and occur on average one or two times per day. a. vestibular neuronitis b. acute labyrinthitis c. BPPV d. meniere disease

C - BPPV: brief episodes that last 2-10 seconds; usually caused by turning the head; caused by the formation of crystalline debris in the semicircular canals that leads to labyrinthine irritation, causing vertigo and nystagmus - Not Meniere's: longer duration of the disease - Not vestibular neuritis: acute, rapid onset - Not acute labyrinthitis: this disorder has hearing loss; acute, rapid onset

Your patient is a 21 year-old male that presents with redness to right eye, photophobia, and tearing. Your patient intraocular pressure is 18 (normal 12-22 mm Hg). Slit lamp exam and staining are normal. The patient is not a contact lens wearer. When the patient was anesthetized with alacaine he still has pain. Which of the following is the most likely diagnosis? a. conjunctivitis b. corneal abrasion c. iritis d. glaucoma e. cataracts

C - Iritis presents as a red eye, photophobia, and tearing. Iritis is often not improved when the eyes are anesthetized with alacaine. A common treatment is eye drops that ease the inflammation. If uveitis is a result of an infection, antibiotics or antiviral medication may be prescribed. - Not Glaucoma due to normal IOP - Not Cataracts: painless - Not Corneal abrasion: not a contact lens wearer - Not Conjunctivitis: viral = acute, bacterial = usually caused by contacts, allergic = no allergies mentioned, toxic = offending agent not mentioned

A 54 year old male presents with fever, trismus, dysphagia, and a sore throat. Which of the following is the most likely diagnosis? a. Parotitis b. Sialadenitis c. Peritonsillar abscess d. GERD e. Sialolithiasis

C - Peritonsillar abscess present with trismus, fever, sore throat and hot potato voice. Peritonsillar abscess are typicaly caused by Group A Strep. They can compromise the upper airway. Empiric treatment is with Unasyn or Clindamycin. Ultimate treatment is incision and drainage. - Sialadenitis, Sialolithiasis, GERD = no fever - Parotitis = no sore throat, dysphagia, and +/- trismus

Your patient is a 54 year old male that presents with erythema and redness to the right submandibular area. There is no pain to palpation over the floor of the mouth. Physical exam of the mouth reveals pus covering from the salivary duct. There is no dental pain or trismus. Which of the following is not true regarding the patient's condition? a. usually resolves in 7-10 days w/abx b. typically responds to Keflex or dicloxacillin c. if the dx is in question an US of the neck is the best imaging modality d. can be caused by salivary duct stone

C - The imaging modality of choice for sialadenitis is a CT scan of soft tissue neck with IV contrast.

Your patient is a 46 year-old male that presents with a piece of metal on cornea with a rust ring that is approximately twelve hours old. Which of the following is not an acceptable method to attempt removal? a. 25 gauge needle b. cotton swab c. magnet d. ophthoburr

C - not acceptable bc you could harm the rest of the eye by having using magnetic force - ophthoburr: tool used to remove FB's from the eye - other two are acceptable

A 19 year-old college student complains of a sore throat for over a week, with fever and general malaise. On exam T-38°C P-70/minute R-20/minute BP-110/76 mmHg. The patient is alert and oriented x 3. The skin is warm, dry, and without rash. The TMs have a normal light reflex and the canals are clear. The oropharynx is inflamed, with bilaterally enlarged tonsils, and a small amount of exudate. The neck is supple, with anterior cervical adenopathy. The lungs are clear. The heart has a regular rhythm without murmurs. The abdomen is soft, nontender and a spleen tip is palpable. The labs reveal a negative rapid strep screen and positive Monospot. The WBC count is 9,000/microliter with a differential of 40% atypical lymphocytes, 35% lymphocytes, 5% monocytes, 10% eosinophils, and 10% neutrophils. Which of the following is the most appropriate treatment? A Penicillin B Erythromycin C Acetaminophen D Acyclovir

C. Acetaminophen

Which of the following is most helpful in the diagnosis of a retropharyngeal abscess? A CBC with differential B fever and a muffled voice on examination C CT of the neck with contrast D history of a recent throat infection

C. CT of the neck with contrast

A 26 year-old male presents with headache, sinus pressure, and sinus congestion for over a month. He has a thick nasal discharge in the mornings, but this improves as the day goes on. He is afebrile. On exam, there is tenderness over the face. TMs have normal light reflex. Nasal mucosa reveals thick yellowish discharge. Neck is supple, without lymphadenopathy. Which of the following is the diagnostic study of choice? A transillumination of sinuses B routine sinus films C CT scan of sinuses D nasal culture

C. CT scan of sinuses

A 35 year-old patient has recurrent seasonal rhinitis and a history of mild asthma. Which of the following should be included for first-line management? A Immunotherapy B Decongestants C Corticosteroid inhalers D Cromolyn sodium (Intal)

C. Corticosteroid inhalers

Small grayish vesicles and punched-out ulcers in the posterior pharynx in a child with pharyngitis is representative of which organism? A Epstein-Barr virus B Group C Streptococcus C Coxsackievirus D Gonorrhea

C. Coxsackievirus

Which of the following disorders is characterized by a "thumb sign" on a soft tissue neck x-ray? a. Laryngitis b. Croup c. Epiglottis d. Foreign Body e. Acute Tonsilitis

C. Epiglottitis is characterized by a thumb sign. - Croup has a steeple sign - Laryngitis & Acute tonsilitis = wouldn't take an X-ray - Foreign body = pt would probs be aspirating

A 29-year-old man presents to the emergency department with bilateral eye pain. The patient states it has slowly been worsening over the past 48 hours. He admits to going out this past weekend and drinking large amounts of alcohol and having unprotected sex but cannot recall a predisposing event. The patient's vitals are within normal limits. Physical exam is notable for bilateral painful and red eyes with opacification and ulceration of each cornea. The patient's contact lenses are removed and a slit lamp exam is performed and shows bilateral corneal ulceration. Which of the following is the best treatment for this patient? A Acyclovir B Erythromycin ointment C Gatifloxacin eye drops D Intravitreal vancomycin and ceftazidime E Topical dexamethasone and refrain from wearing contacts

C. Gatifloxacin eye drops

A 4 year-old boy presents with pain and irritation of his left ear. Otoscopic examination reveals an insect in the left auditory canal. The tympanic membrane is not completely visualized. Which of the following is the most appropriate management of this patient? A. Debrox insertion with suction removal B. Irrigation with room temperature saline C. Insertion of 2% lidocaine solution with suction or forceps removal D. Polymyxin drop insertion via wick

C. Insertion of 2% lidocaine solution with suction or forceps removal

What is the most common location of anterior nasal epistaxis? A Middle turbinate B Posterior ethmoid artery C Kiesselbach's triangle D Inferior turbinate

C. Kiesselbach's triangle

A 22 year-old woman presents with sneezing, runny nose, postnasal drip, and nasal congestion for the last week. She says this happens every spring. She is not allergic to any medications. Which of the following is the most appropriate pharmacologic treatment for this patient? A Azithromycin (Zithromax) B Phenylephrine (Neo-synephrine) C Nedocromil D Pseudoephedrine

C. Nedocromil

A 54 year-old type 2 diabetic male presents for follow up evaluation of previously diagnosed persistent otitis externa. Early in the disease process, a CT scan was obtained secondary to non-improvement on antibiotics. Results showed osseous erosion of the floor of the ear canal. He has been on ciprofloxacin 1000 mg twice daily for two months since the CT scan and currently has no further edema, erythema, or exudate from the external auditory canal or surrounding tissue. Which of the following is an appropriate treatment plan? A. Continue prophylactic antibiotics for an additional 6 weeks B. Immediately discontinue antibiotics C. Obtain gallium scan to ensure reduction of inflammatory process D. Skin swab culture of healthy tissue

C. Obtain gallium scan to ensure reduction of inflammatory process

A 13 year-old presents with pain in his right ear and loss of hearing since yesterday. He has never had an episode like this before. On exam vital signs are T- 38°C P- 70/minute R- 18/minute BP- 90/60 mmHg. Neck is supple, without lymphadenopathy. Right tympanic membrane is not visible; the canal is swollen, with small amount of exudate and blood noted. There is tenderness of the external ear, especially with gentle traction of the tragus. Left tympanic membrane is normal, and the canal is clear. Oropharynx is normal. Which of the following is the most appropriate topical treatment for this patient? A Tobramycin otic drops B Erythromycin solution C Offloxacin otic drops D Gentamicin drops

C. Offloxacin otic drops

A 60-year-old male presents with complaints of irritation and a white plaque on his tongue. He denies pain. During physical exam you are unable to remove the white plaque from the mucosa with a tongue depressor. What is the most likely diagnosis, represented as follows? a. epiglottis b. oral herpes simplex c. oral leukoplakia d. parotitis e. aphthous ulcers

C. Oral leukoplakia cannot be removed from the mucosa using a tongue depressor like oral thrush can. It is considered a premalignant lesion and the chance of transformation into oral squamous cell carcinoma is 0-20%. The vast majority will not turn malignant. The condition is more common in smokers and people should be advised to stop smoking and limit alcohol consumption to reduce their risk. Management usually involves regular review of the lesion to detect any possible malignant change early, and thereby significantly improve the prognosis, which normally is relatively poor for oral squamous cell carcinoma

A 20 year-old woman presents with a 3-day history of sneezing, watery nasal discharge, and a nonproductive cough. Her throat was sore for the first 2 days, and she now complains of fatigue and difficulty breathing because of her "stuffy nose." Which of the following is most likely to improve this patient's status? A Ascorbic acid B Amoxicillin C Pseudoephedrine D Chlorpheniramine

C. Pseudoephedrine

Which of the following is the most common organism causing mastoiditis? a. H. influenza b. S. aureus c. S. pneumoniae d. Group B Hemolytic strep e. Strep pyogenes

C. S. pnuemoniae Streptococcus pneumoniae is the most common organism that causes mastoiditis. Mastoiditis is the most common suppurative complication of otitis media. Pseudomonas is another consideration especially on recurrent infections with a ruptured tympanic membrane.

A patient with type 2 diabetes mellitus presents for a yearly eye exam. Ophthalmoscopic exam reveals neovascularization. Which of the following is the most likely complication related to this finding? A. Glaucoma B. Cataracts C. Vitreous hemorrhage D. Optic Neuritis

C. Vitreous hemorrhage

Which of the following have not been identified as a risk factor for barotrauma to the tympanic membrane? a. diving b. flying c. allergic rhinitis d. blast injuries e. driving at high elevations

C. allergic rhinitis Diving, flying, and blast injuries are identified as circumstances predisposing patients to barotrauma. Barotrauma is caused by pressure difference between the outside world and the inside of the middle ear. Allergic rhinitis can cause eustachian tube dysfunction but usually does not cause barotrauma. There usually is significant pressure difference between the middle ear and the outside world when the patient is exposed to pressure differences of diving, flying or blast injuries.

A 7 year old boy presents to your clinic with thick purulent discharge from her right eye which was significantly worse this morning. What should you do next? A. take a sample of the discharge and send it to the lab for culture. B. send the patient home to perform warm compresses for 20 minutes 3 times daily tell them to come back if symptoms become worse C. moxifloxacin 0.5% drops tid for 7 to 10 days D. topical 1% prednisolone acetate qid

C. moxifloxacin 0.5% drops tid for 7 to 10 days

What nerve does HSV1 or HSV2 lay dormant, and when actives results in herpes simplex keratitis?

CNV (trigeminal)

An old woman with diabetes comes in complaining that the outside of her ear hurts really badly when she lies down on it or cleans her ears with a Q-tip. Otoscopy reveals granulation, necrosis and redness to the EAC. Which test do you order to confirm malignant otitis externa? Which test is most accurate? What is the treatment?

CT/MRI to confirm Bx is most accurate admit, IV cipro

A 27-year-old man presents following an incident where he was struck in the left eye with a paint ball. He notices a sudden decrease in vision in the left eye, from 20/20 before the accident, to counting-fingers vision after the accident. On exam, the left pupil appears whitish, and visual acuity is greatly decreased. The patient does not have any history of other medical problems. On dilated eye exam, the lens in the left eye appears whitish anteriorly, with a spoke-like pattern. On direct ophthalmoscopy, the red reflex is diminished and retinal details are indistinct.

Cataract

A 65-year-old man presents complaining of generally decreased vision and difficulty driving at night due to glare from oncoming headlights. He describes having trouble reading the small print on his television screen. He is healthy and has no history of any other eye problems. His best corrected visual acuity is noted to be 20/50 in the right eye and 20/40 in the left eye. On exam, a yellowish opacification of the lens in the left eye is noted. On ophthalmoscopy, the red reflex in the left eye is obscured centrally, and the details of the fundus are indistinct. No other abnormalities are found.

Cataract

A 66 year-old male presents complaining of 6 month history of progressive blurred vision without associated pain. On examination there is no erythema or injection of the sclera. On funduscopic examination there is an absent red reflex and a cloudy lens. What is the most likely diagnosis?

Cataracts

a 78-year-old man complains of slowly progressive vision loss over the last several years. He describes his vision as if he is looking through "dirty glass" and reports seeing a white halo around lights. On physical exam, there is clouding of the lens and no red reflex. What is the dx?

Cataracts

A 65 year old male construction worker comes in because he noticed his eye-sight is getting very poor. There is no pain. He smoked for many years and has always worked outside. PE reveals an absent red light reflex and opaque lens. He can still see and perform his ADLs. What is it and what is the treatment at this point?

Cataracts observation

A 72 y/o male presents with unilateral vision loss. Patient has a history of HTN and DM. On exam, the R pupil has relative afferent pupillary defect. On otoscope exam you note blood and thunder fundus. What is the appropriate dx.

Central Retinal Vein Occlusion

is characterized by a sudden, painless vision loss. A cherry red spot is characteristic on the macula, along with pallor to the retina

Central retinal artery occlusion

What is the ddx for continuous & nonfatiguable Vertical nystagmus -gait abnormalities -gradual onset -other + CNS signs? What is the category? What is affected?

Cerebellopontine tumors-Migraine-Cerebral vascular dz-MS-Vestibular Neuroma brainstem or cerebellum

a 25-year-old patient presents with unilateral hearing loss. Weber reveals lateralization to the right ear. Rinne test reveals the following: RIGHT: bone conduction = 10 seconds, air conduction = 5 seconds; LEFT: bone conduction = 5 seconds, air conduction = 10 seconds. DX? TX?

Cerumen impaction -Irrigation, curette/otoloop -Don't use q-tips

A surfer comes in complaining of yellowish d/c coming out his his left ear. He says that it "smells really bad." He's had bouts of ringing in this ear, spinning feelings and hearing loss. What is it? What is the treatment?

Cholesteatoma surgery and ossicle reconstruction

43-year-old male with a "lifelong" history of chronic ear infections and episodic purulent drainage from his right ear canal. The patient currently is without symptoms. Examination of the ear shows a clear external canal, but the tympanic membrane is retracted and there is a pocket of white material and an opacity of the pars flaccida. The Weber test lateralizes to the right and Rinne shows air conduction > bone conduction on the left and bone conduction > air conduction on the right. Preparations are made to undergo a non-contrast computed tomography (CT) scan of the temporal bone. Dx: Tx:

Cholesteatoma - Surgical excision

A 6 year old boy comes in with his mother because his right ear is having d/c and he has been complaining of hearing problems out of his right ear for >6 weeks. PE reveals a perforated TM. What is it? What type of hearing loss? What is the treatment?

Chronic otitis media CHL remove infected debris + oflo or cipro TM repair/reconstruction

A 47 year old man comes in with pressure in his face. He has been dealing with this for 4 months, but hasn't been able to get off of work as a welder to come in and "get looked at." He spends much of the day leaning forward, which has made things worse and now he has purulent nasal d/c. PE reveals a fever and congestion. What is it? What is needed for Dx? MC tx? Referral?

Chronic sinusitis Bx or histology combo nasal irrigation + topical/oral glucocorticoids ENT f/u

a 10-year-old boy who was hit in the right eye with a piece of bark that was thrown on the playground. He developed sudden onset of eye pain, photophobia, tearing, and blurring of vision. He claims there is "something stuck in my eye." On physical examination, there is significant conjunctival injection. DX? TX?

Corneal Abrasion -Refer if deep -Topical abx (oflaxacin, -educate on safaty glasses use

A 22-year-old woman presents to the ER with acute bilateral eye pain after removing her contact lenses 1 hour ago. She reports forgetting to remove her daily wear lenses at bedtime and awakening in the morning with bilateral mild eye pain. On removal of both lenses she reports markedly increased pain, photophobia, and foreign body sensation. With glasses, her visual acuity is 20/30 bilaterally. On slit lamp exam, she has no ulceration but diffuse fluorescein uptake in a contact lens distribution bilaterally.

Corneal abrasion

A 32-year-old man is struck in the face with a branch while hiking through dense bush. He is not wearing any eye protection and feels immediate pain, discomfort, and watering on opening his right eye. He does not wear contact lenses. He has photophobia in the right eye, and there is marked conjunctival injection but normal visual acuity. After instillation of a topical anesthetic his symptoms resolve, facilitating a complete slit lamp exam, which reveals a 3 mm corneal defect with fluorescein uptake without ulceration or any foreign body.

Corneal abrasion

a 34-year-old contact lens wearer with severe pain, redness, and photophobia. Eyes are injected with cloudy discharge unilaterally. A dense corneal infiltrate is visible with fluorescein staining. DX: TX:

Cornel Ulcer -Referral -Avoid contacts -Antibiotics: Ciprofloxacin, ofloxacin, gentamicin, erythromycin, polytrim, tobramycin

Diphtheria

Corynebacterium diphtheriae produces ​exotoxin → myocarditis & neuropathy

Your patient is a 23-year-old female who is a contact lens wearer. She presents with an injected right conjunctiva. She has no foreign body sensation or photophobia. There is no uptake of the stain and slit lamp exam is otherwise normal. Which eye drops are the most ideal for this patient. a. sulfacetamide b. gentamicin c. zithromax d. ciloxan e. vancomycin

D - Contact lens wearers are prone to getting bacterial infections (bacterial conjunctivitis) especially pseudomonas of the eye. Ciloxan has the best pseudomonas coverage. Ciloxan = Ciprofloxacin

Your patient is a 34 year-old male that presents with right eye pain, photophobia, and a scratch of the cornea with staining. There was no hyphema. Which of the following is not part of the treatment of this condition? a. gentamicin eye drops b. oral analgesics c. sunglasses d. patching

D - Eye patching is no longer recommended for corneal abrasions. A meta-analysis of five randomized controlled trials (RCTs) failed to reveal an increase in healing rate or improvement on a pain scale. Two subsequent RCTs (one in children, one in adults) reported similar results. In the past, patching was thought to reduce pain by reducing blinking and decreasing eyelid-induced trauma to the damaged cornea. However, the patch itself was the main cause of pain in 48 percent of patients. Children with patches had greater difficulty walking than those without patches. Furthermore, patching can result in decreased oxygen delivery, increased moisture, and a higher chance of infection. Thus, patching may actually retard the healing process

Your patient is a 45 year old male that presents with episodic vertigo, hearing loss, and tinnitus over the last year. His Brain MRI was unremarkable. He has had no preceding viral syndrome. Which of the following is the most likely diagnosis? a. BPPV b. Vestibular Neuritis c. Labyrinthitis d. Meniere's Disease e. Vestibular Schwannoma

D - Meniere's = 3 triad of symptoms (vertigo, hearing loss, and tinnitus); BPPV doesn't have tinnitus and has shorter duration of vertigo; Meniere's = longer duration of vertigo Vestibular Neuritis = no ear symptoms Labyrinthitis & VN = due to URI Vestibular Schwannoma = Brain MRI was unremarkable

Your patient is a 64-year-old female that presents with gradual onset of peripheral bilateral vision loss over the last year. Which of the following is the most likely diagnosis is the most likely diagnose? a. strabismus b. optic neuritis c. macular degeneration d. open angle glaucoma e. closed angle glaucoma

D - Open angle glaucoma is an optic neuropathy that has a progressive peripheral visual field loss and is eventually followed by central visual loss. Note: both optic neuritis and macular degeneration cause central vision loss. Strabismus is a misalignment of the eyes. Not closed angle glaucoma bc it is a gradual onset (closed = sudden --emergency)

Your patient is a 45 year old female that presents with a sudden onset of visual loss in the left eye. Funduscopic exam reveals hyperemia and swelling and blurring of the optic disc. There is no history of headache. The patient's other eye is normal. The patient describes a scotoma in the central vision of his affected eye. Based upon this data, which of the following is the most likely diagnosis? a. pseudotumor cerebri b. ophthalmic migraine c. retinal detachment d. optic neuritis

D - Optic neuritis; Pseudotumor cerebri usually affects both eyes and is accompanied with a headache. Ophthalmic migraine usually has headaches with it and will not have any optic disc changes on exam. Retinal detachment does not have these visual field changes and will not have these optic disc changes on physical exam. - optic neuritis bc swelling of optic disc, central scotoma/blind spot; unilateral

All of the following are potential etiologies for parotitis except? a. Stensen's Duct obstructed w/ a stone b. Retrograde salivary stasis and seeding in Stensen's duct c. Staphylococcus auerus d. Streptococcus Virdans

D - Potential etiologies of parotitis include: anything that obstructs Stensen's Duct (Stone or Tumor), poor oral hygiene, dehydration, retrograde salivary stasis and seeding of Stensen's Duct with oral flora, viruses, or staphylococcus aureus.

(on test???) Which of the following bacteria is not a possible cause of bacterial tonsillitis? a. Group A Streptococcus b. Neisseria Gonorrhea c. Mycoplasma d. Proteus e. Chlamydia

D - Proteus; All of the following are possible causes of bacterial tonsillitis: Group A Streptococcus, Neisseria Gonorrhea, Mycoplasma, Chlamydia, Tularemia, and Group C and Group G Strep. Group C and Group G Strep do not cause rheumatic fever. Viral etiologies include CMV, HSV, Mononucleosis, and Influenza

A 23-year-old woman comes to your office with a 6-month history of dizziness. She "feels dizzy" when she stands up (as if she is going to faint). The sensation disappears within a minute. She has a history of major depression. She started taking doxepin 6 months ago, and her depression has improved much since that time. The patient's blood pressure is 140/ 90 mm Hg sitting and decreases to 90/ 70 mm Hg when she stands. There is no ataxia, nystagmus, or other symptoms. What is the most likely diagnosis in this patient? a. vestibular neuritis b. acute labyrinthitis c. BPPV d. orthostatic hypotension *note: not really in our notes

D - This patient has orthostatic hypotension. This case illustrates the importance of obtaining an accurate history in the patient who complains of "feeling dizzy." In any patient who has this complaint, it is important to ask four specific questions: (1) Can you describe your dizziness? (2) How much of the dizziness is a sensation of the "world spinning around you," and how much would be a sensation of "things going black in front of you and a feeling that you're about to pass out"? (3) How long does the feeling of dizziness last— seconds, minutes, or hours? and (4) Are there any other symptoms present when you feel dizzy, such as deafness, ear fullness, or ringing in the ears? The answers to these questions are the most important clues to leading you toward a diagnosis. This patient does not have clear symptoms of vertigo, and one should focus on non- vertigo-related dizziness. Orthostatic hypotension is typically initiated after standing up suddenly. There is a transient lack of cerebral blood flow. The feeling described is that of a subjective dizziness, and the patient feels as if he or she might faint. It is not associated with any other neurologic sensations or ear symptoms. It is also characterized by a lack of symptoms when sitting or lying. Medications are the most likely culprits in this scenario, but occasionally there is no clear etiology for the autonomic disturbance. In this clinical case problem, the orthostatic hypotension is almost certainly associated with the beginning of the tricyclic antidepressant therapy 6 months ago.

Which physical examination finding distinguishes allergic rhinitis from other rhinitis etiologies? A Clear rhinorrhea B Erythematous pharynx C Nasal flaring D Pale nasal turbinates

D Pale nasal turbinates On physical examination, the mucosa of the turbinates is usually pale or violaceous with allergic rhinitis because of venous engorgement in contrast to the erythema of viral rhinitis.

Your patient is a 39-year-old second grade teacher who states that she developed acute hoarseness 3 days ago. Prior to that, she had a cold, the symptoms of which are improving. There is no history of smoking or other tobacco use. What is the most important intervention for the patient at this time? A Discuss that due to her occupation, she is at increased risk of leukoplakia of the vocal cords B She should be placed on an antibiotic as she most likely has a bacterial infection C Discuss her increased risk of vocal cord paralysis D Advise the patient to avoid singing or shouting until her normal voice returns

D. Advise the patient to avoid singing or shouting until her normal voice returns

A 2 year-old female presents with purulent nasal discharge bilaterally with fever and cough for several days. Her mom had taken her out of daycare for a similar occurrence 2 months ago, that was treated with Amoxicillin. Exam further reveals halitosis and periorbital edema. Treatment should be initiated with which of the following? A Antihistamines B Ribavirin (Rebetol) C Intranasal corticosteroids D Amoxicillin-clavulanate (Augmentin)

D. Amoxicillin-clavulanate (Augmentin)

Your patient is a 34-year-old male that has had maxillary sinus pressure and purulent drainage for 3 weeks. The patient has had a history of anaphylaxis with penicillin. Which of the following is the most appropriate antibiotic choice? a. Augmentin b. Cephalexin c. Amoxicillin d. Bactrim e. Ampicillin-Sublactam IV

D. Bactrim - Acute bacterial (rhino)sinusitis The major organisms associated with bacterial acute sinusitis is streptococcus pneumonia, H. influenzae, and M. Catarrhalis. With the patients history of anaphylaxis with penicillin, the best choice is D bactrim Note: A, C, E = penicillins and Cephalexin is a cephalosporin that can have a 10% cross-reactivity with PCN-allergic pts. (therefore, avoid in pts with anaphylaxis rxns to PCN)

Which of the following areas is the most common site for epistaxis to occur? a. arterioles off the external carotid artery b. arterioles off the internal carotid artery c. Stensen's Plexus d. Kiesselbach's Plexus e. sphenopalatine arteries ??????????????????

D. Kiesselbach's Plexus - Ninety percent of nose bleeds arising from Kiesselbach's Plexus. - Stensen's Plexus does not exist in the nasal cavity, it is a duct that drains the parotid glands into the oral cavity - Sphenopalatine arteries = not as common and is more serious - The tributaries of the internal carotid artery do supply blood to the posterior nasopharynx and can be the source of some arterial bleeds.

A 52 year-old female presents with complaints of intermittent episodes of dizziness, tinnitus, and hearing loss in the right ear for 6 months. She describes the dizziness as the "room spinning around her," with the episodes typically lasting for 2 to 4 hours. Physical examination reveals horizontal nystagmus and right ear hearing loss, but the remainder of the examination is unremarkable. Which of the following is the most likely diagnosis? A Acute labyrinthitis B Positional vertigo C Acoustic neuroma D Ménière's syndrome

D. Ménière's syndrome

A 12 year-old presents with complaint of both eyes "watering." He also complains of sinus congestion and sneezing for two weeks. On exam vital signs are T-38°C, P-80/minute, and RR-20/minute. The eyes reveal mild conjunctival injection bilaterally, clear watery discharge, and no matting. Pupils are equal, round, and reactive to light and accommodation. The extraocular movements are intact. The funduscopic exam shows normal disc and vessels. The TMs are normal and the canals are clear. The nasal mucosa is boggy, with clear rhinorrhea. Which of the following is the most helpful pharmacologic agent? A. Artificial tears B. Tobramycin drops C. Erythromycin ointment D. Naphazoline (nacphcon-A) drops

D. Naphazoline (nacphcon-A) drops

Which physical examination finding distinguishes allergic rhinitis from other rhinitis etiologies? A Clear rhinorrhea B Erythematous pharynx C Nasal flaring D Pale nasal turbinates

D. Pale nasal turbinates

An 18-year-old sexually active female was seen in the student health clinic 1 week ago for a sore throat. A streptococcal antigen test was positive, and she was given a prescription for oral penicillin. After 3 days, she stopped her medication because she felt better. She now presents with a severe sore throat. On physical examination, she has a temperature of 102.6° F (39.2° C), marked pharyngeal erythema, medial deviation of the soft palate on the left, tender left anterior cervical adenopathy, and a "hot potato" voice. The rest of her history and physical examination are unremarkable. Which of the following is the most likely diagnosis? A Recurrent streptococcal pharyngitis B Infectious mononucleosis C Gonococcal pharyngitis D Peritonsillar abscess

D. Peritonsillar abscess

An elderly patient with a history of hypertension presents with epistaxis. On examination you note blood from both nares and down the posterior oropharynx. Examination of the nasal cavity with an ENT headlamp does not show an area of bleeding. Which of the following is the treatment of choice in this patient? A Electrical cautery B Direct pressure on the nose C Petroleum jelly application D Posterior nasal packing

D. Posterior nasal packing

A 3 year-old patient is brought in with a 10 day history of clear nasal drainage and cough which has now developed into otalgia and fever exceeding 101 degrees F for the last 5 days. Mom denies other chronic medical problems. The patient has had similar complaints three times in her life. Which of the following is the most likely causative organism? A Mycoplasma pneumoniae B Pneumocystis jiroveci C Pseudomonas aeruginosa D Streptococcus pneumoniae

D. Streptococcus pneumoniae

Which of the following is the most likely organism in a 2 year-old child with acute otitis media? A Staphylococcus aureus B Moraxella catarrhalis C Pseudomonas aeruginosa D Streptococcus pneumoniae

D. Streptococcus pneumoniae

A 9 year-old patient presents with conjunctivitis after swimming at the local pool. On examination, there is visible lid edema with redness of the palpebral conjunctiva, copious watery discharge, and scanty exudate. The sanitation system of the public pool is through the use of a salt water system; therefore, the possibility of a chemical induced conjunctivitis is almost non-existent. Which of the following should be instituted to prevent the sequalae of the condition A. Ketorolac tromethamine (Acular) B. Dexamethasone ophthalmic C. Naphazoline HCL (Naphcon A) D. Sulfacetamide ophthalmic

D. Sulfacetamide ophthalmic

Which of the following clinical findings differentiates periorbital from orbital cellulitis? A. erythema B. fever C. lid edema D. worsening pain with eye movements E. development of a rash on the face

D. worsening pain with eye movements

a 64-year-old diabetic patient who is being seen for a routine health screening. On fundocopic exam, you see cotton wool spots, hard exudates, blot and dot hemorrhages, neovascularization, flame hemorrhages, A/V nicking. What is the most likely dx?

DM retinopathy

A patient comes in with a swollen, red bump between her eye and nose. There is a huge pustule with purulent d/c on the tip of it and it looks infected. What is it? What are the bugs? what is the tx?

Dacrocystitis SA/SE, GABS, pseudo give a compress and clindamycin (or vanco + ceftriaxone)

a 12-year-old with severe unilateral right eye pain and pressure. On physical exam there is swelling, redness, tearing and drainage from the outermost part of the affected right eye. DX? TX?

Dacryoadenitis -Warm compress,& rest -I&D if needed -Tetracycline (if bad)

A patient comes in complaining of severe tooth pain, and temperature sensitivity. On exam you note difficulty opening the mouth, lymphadenopathy, and a fluctuant mass. Dx: Tx:

Dental Abscess -Referal -I&D -PO Augmentin or clindamycin -NSAIDS -Root canal

An elderly woman comes in c/o a blind spot in her field of view. "Lines seem curvy and everything looks small." Fundoscopy reveals drusen bodies and Amsler grid is positive. What is it? what is the treatment? What is inevitable

Dry macular degeneration zinc, vit C/E Amsler grid monitoring legal blindness

A pt presents with otorrhea, tenderness, crusting, and **HL, what does this pt have? a. cerumen impaction b. malignant otitis externa c. CSOM d. AOM e. otitis externa

E

A 38-year-old woman comes to your office with a 1-year history of episodic dizziness, ringing in both ears, a feeling of aural fullness, and hearing loss. The symptoms come on every 1 or 2 weeks and usually last for 12 hours. Nausea and vomiting are present. When asked to describe the dizziness, the patient says, "The world is spinning around me." On physical examination, the patient has horizontal nystagmus. The slow phase of the nystagmus is to the left, and the rapid phase is to the right. Audiograms reveal bilateral sensorineural hearing loss in the low frequencies. What is the most likely diagnosis in this patient? a. vestibular neuritis b. acute labyrinthitis c. positional paroxysmal vertigo d. orthostatic hypotension e. Meniere disease

E. Meniere disease. The classic features of Meniere disease are recurrent episodes of vertigo, fluctuating low-frequency sensorineural hearing loss, tinnitus (ringing or buzzing in the ears), and aural fullness in the affected ear. The vertigo typically lasts hours, not minutes or days. The fluctuating hearing may not be related temporally to the vertigo. For the diagnosis of Meniere disease, the characteristic pattern of vertigo lasting a matter of hours as well as sensorineural hearing loss must be present. One additional factor (aural fullness or buzzing tinnitus) should also be present.

a 72-year-old with complaints of dry eyes coupled with excessive tearing. On exam the conjunctiva appear red and the left eyelid is turned outward. DX? TX?

Ectropion - eye lubricating drops -Surgery

a 14-year-old who is brought to your Emergency Department (ED) with an intractable nosebleed. Pinching of the nose has failed to stop the bleed. In the ED a topical vasoconstrictor is tried but also fails to stop the bleeding. Dx: Tx:

Episaxis -Apply direct pressure & Lean forward -Anterior nasal packing -Topical decongestants (cocaine, phenylephrine)

Patient will present as → a 14-year-old who is brought to your Emergency Department (ED) with an intractable nosebleed. Pinching of the nose for a few minutes has failed to stop the bleed. In the ED a topical vasoconstrictor is tried but also fails to stop the bleeding.

Epistaxis

A woman comes in c/o hearing loss and ringing in her right ear after getting over a sinus infection. She c/o of feeling "off balance," her ear keeps popping and feelslike she is swimming underwater. Weber testing lateralizes to her right ear and Rinne reveals BC>AC. Otoscopy is normal. What is it? What is the treatment?

Eustachian tube dysfunction Autoinsufflate and decongestants

Name the clinical signs in the modified centor criteria

Fever (1) Tonsillar Exudate (1) Absent Cough (1) Anterior Cervical LAD (1) Age 3-14 (1) Age > 44 (-1)

A 5-year-old child with no known drug allergies is diagnosed in your clinic with bilateral acute otitis media. Which of the following is the drug of choice?

First choice antibiotic treatment for acute otitis media includes a 10-day course of amoxicillin (80 to 90 mg/kg/day in two divided doses) or a combination of erythromycin (50 mg/kg/day) and a sulfonamide (150 mg/kg/day). Reasons for amoxicillin therapy include spectrum of activity including both susceptible and intermediate resistant S pneumoniae, safety, cost, and tolerability.

a 25-year-old HIV positive male with pain associated with his gums. He has also noted bleeding of his gums when he brushes his teeth. On physical exam, you observe a bright erythematous line along the gingival margin. dx: tx:

Gingivitis -Dental referral -Dental hygiene -PNC (maybe ) -Peroxide rinse

Subjective tinnitus is most commonly accompanies by?

Hearing loss

A 41 year old female comes into the clinic with a lesion on her lip. You notice grouped vesicles on an erythematous base. She said she "made out with a rando a few days ago and thinks he had some kind of pimple-looking thing on his lips." Before the lesion appeared, the area was itchy and felt like a "burning tingle." What is it What Dx tests can be ordered? How is it treated?

Herpes 1 PCR-TOC HSV-1 serology (G.S.) Cx Tzanck smear oral valacyclovir 2g BID qd

A patient comes in complaining of vision changes in one eye and redness/pain out of no where. The cornea appears hazy and there is preauricular LAD. Staining reveals a dendritic corneal ulcer. What is it and how is it treated?

Herpes Keratitis top antivirals Trifluridine gangciclovir ung Or PO acyclovir

A 44 year old female comes in with irritation, light sensitivity and a mild conjunctival injection. On fluorescein exam you observe a dendritic lesion. DX? TX?

Herpes Simplex Keratitis -Refer -Debridement & patching -No steroids -Topical antiviral (idouridine, vidarabine, & trifluridine )

a 37-year-old female with an intense, tearing pain in her right eye. She was recently placed on topical corticosteroids for suspected allergic conjunctivitis. On visual inspection the conjunctiva appears red. A fluorescein stain of the eye exhibits a shallow ulcer with a dendritic appearance and irregular borders. DX? TX?

Herpes Simplex Keratitis -Referral -Topical antiviral (idoxuridine, vidarabine, and trifluridine) -debridement & patching

A patient comes in complaining of a fever, headache, drooping, and difficulty swallowing. On exam you note painful burning lesions, cervical lymphadenopathy and small vessels in the back of the throat. Dx: Tx:

Herpetic Stomatitis - Self limiting -Analgesia -Hydration -Acyclovir -Viscous lidocaine

a 15-year-old male with pain, redness, and swelling of the upper eyelid for the last 3 days. There are no visual changes or photophobia. Examination reveals a tender, erythematous, and outward-pointing edema of the right eyelid. DX? TX?

Hordeolum -Warm compress -I&D (if persistent 48 hrs) -Topical Abx (Fluoquinilines, polymix/trimethoprim)

A 30-year-old man presents with a painful, swollen right eye for the past day. He reports minor pain on palpation of the eyelid and denies any history of trauma, crusting, or change in vision. He has no history of allergies or any eye conditions and denies the use of any new soaps, lotions, or creams. On exam, he has localized tenderness to palpation and erythema on the midline of the lower eyelid near the lid margin. The remainder of the physical exam, including the globe, is normal.

Hordeolum (Stye) & chalazion

MRI to confirm diagnosis ENT referral Observation Surgical excision

How would you treat a patient with Acoustic neuroma?

Diagnose with Dix-Hallpike maneuver Treat with Epley maneuver and Meclizine

How would you treat a patient with BPPV?

ENT referral Low salt diet Diuretics

How would you treat a patient with Meniere's Disease?

ENT referral Hearing aids Cochlear implant

How would you treat a patient with Presbycusis?

ENT referral Possible surgery

How would you treat a patient with a cholesteatoma?

ENT referral Observe Surgical reconstruction

How would you treat a patient with a tympanic membrane perforation?

Immediate ENT referral for I&D with bolster

How would you treat a patient with external ear hematoma?

ENT referral PO steroids Spontaneous resolve

How would you treat a patient with idiopathic sudden sensorineural hearing loss?

ENT referral to r/o nerve damage Meclizine

How would you treat a patient with labyrinthitis/ vestibular neuritis?

Immediate ENT referral I&D Abx

How would you treat a patient with mastoiditis?

Chronic= Debride and topical steroids Acute= Debride, topical steroids, topical abx

How would you treat a patient with otitis externa?

Observe Amoxicillin 7-10 days If chronic, ENT referral for TM tubes

How would you treat a patient with otitis media?

a 14-year-old who sustained a blunt trauma to his right eye after being struck by a baseball. He complains of blurry vision. On physical exam, you note unequal pupils, injected conjunctiva/sclera, and blood in the anterior chamber of the right eye. DX TX

Hyphema -Refer -Admit & monitor -Rest

How would you treat a patient with septal hematomas?

I&D within 3-5 days

What is the most common cause of tinnitus? Most common sound?

Idiopathic high pitched, continuous tones

a 23-year-old female PA student with hearing loss and tinnitus that began yesterday. She describes a sensation of the room "spinning" around her. She feels extremely nauseous and vomited already one time this morning. On physical exam, a horizontal nystagmus is observed. You excuse her from her histology exam later that afternoon. Dx: Tx:

Labyrinthitis - Referral -CT/MRU -Corticosteroids -Will resolve in 7-10 days -Tx: symptomatic (anti-emetics)

An elderly male patient comes in b/c he feels off balance and it's affecting the way he walks. He c/o nausea and that the "room is spinning and it won't stop." PE reveals horizontal nystagmus. The patient is positive for UL HL and tinnitus. What is it? What is first line tx?

Labyrinthitis/Vestibular Neuritis. (The HL/tinnitus indicates labyrinthitis as well) Glucocorticoids

a 27-year-old mezzo-soprano who states that she developed acute hoarseness 2 days ago. Prior to that, she had a cold, the symptoms of which are improving. There is no history of smoking or other tobacco use. She is very worried as she has an upcoming performance 3 days from now. Dx? Tx?

Laryngitis -Rest voice -Do not clear throat -Humidifier -Hydration -Tx underlying condition (URI) -Surgery

A 40 year old male comes in complaining of a fever, and a bilateral budging neck, and restriction of neck movement. The patient stated that he has an URI prior in the week. On exam you note a displaced tongue, dysphonia, trismus, drooling and stridor. On the floor of the mouth you note pus. The patient begins to have difficulty breathing. Dx: Tx:

Ludwig's angina EMERGENCY -Secure Airway -Immediate ENT/ dental eval -I&D -Admit to ICU -IV abx (PNC & flagyl, clindamycin, ampicillin)

A 30 year old woman comes in with painful vision loss in one eye. The pain is worse when she moves her eyes. Hx reveals that she has MS. PE shows a marcus gunn pupil, fundoscopy shows a 2/3 disc to cup ratio. What imaging test should you order b/c of the MS? What is this? What is the tx?

MRI Optic neuritis IV methylprednisolone, then oral CS

a 62-year-old male who arrives for his follow-up visit for chronic central visual loss. He describes a phenomenon of wavy or distorted vision that has deteriorated rather quickly. The patient is frustrated because he "just can't drive anymore" and he is "having difficulty seeing words when he reads." When looking at a specific region of the Amsler grid, he reports a dark "spot" in the center, with bent lines. On the fundoscopic exam, you note areas of retinal depigmentation along with the presence of yellow retinal deposits. what is the DX?

Macular degeneration

A 28 year old male comes into the office complaining of ear pain. He has a PMH of DM. He states that he has had the ear pain for about a week now, and it has gotten worse. There is more white purulent discharge. On exam you note severe otalgia, severe edema of the ear camal and edema in the preauricular soft tissue. Dx? Pathogen? Tx?

Malignant otitis externa -Pseudomonas - IV abx -ENT consilt

a 10-year-old boy with otalgia, worsening over the last 5-days and associated with nasal congestion. The patient is afebrile with a temperature of 98 ° F. Examination reveals edema of the external auditory canal producing an anterior and inferior displacement of the auricle with percussion tenderness posteriorly. Dx & Test: Pathogen : Tx:

Mastoiditis CT scan CBC,ESR, CRP Tympanocentesis -S. Pneumoniae -IV Abx : Cephazolin, Vancomycin -Myringotomy/ T-tubes

A one year old girl is brought into the emergency room after she woke her parents up in the night with screaming. She has a high fever and seems lethargic. The parents say they notice that the child keeps pawing at her ears. Posterior to the right ear is swollen, red and edematous, Otoscopy reveals a bulging tympanic membrane that also appears red. The EAC is very narrow, even with the smallest otoscope cover. What is this? What is first line imaging? What is the treatment?

Mastoiditis! CT scan w/ contrast myringotomy + T-tubes IV vanco + ceftazadime, cefepime or pip-tazo

a 32-year-old male with complaints of recurrent, episodic vertigo lasting up to 8 hours per episode for the past 3 months. The attacks generally last less than half an hour and are associated with decreased low-frequency hearing in the left ear along with nonpulsatile tinnitus in the ipsilateral ear. You obtain an audiogram which shows a low-frequency hearing loss in the left ear only. Dx: Tx:

Meniere's Disease -Symptomatic -TM Tubes (if severe) -Diuretics -Avoid salt, caffeine, cholate, Alcohol

A 42 year old man comes in complaining of vertigo that lasts from 5 minutes to 2 hours daily. He has problems hearing in both ears during these episodes and complains of a wooshing sound in his ear. His ears feel very "full." This combination of sx has caused him to feel sick and throw up a few times a week. PE reveals horizontal nystagmus. What is it? Patient education? Tx if education doesn't work?

Meniere's disease Avoid salt, caffeine, nicotine, chocolate, & ETOH antihistamines, benzos, diuretics

A 7 year old boy comes in to urgent care with blood dripping down from his left nares. His mom complains that his nose smells TERRIBLE and looks gross....He was playing with his sister's beads while she was making a necklace and then all hell broke loose. The little boy is breathing like "the big bad wolf," says his big sister. What is it? What do you use to look in his nose? What is the treatment?

Nasal foreign body Head light and otoscope w/ rigid or flexible fiberoptic endoscopy for direct visualization Remove w/ positive pressure technique or instrument

Patient will present as → a 26-year old female who is being seen for a routine exam is noted to have multiple teardrop-shaped, pale-boggy growths partially obstructing the nasal passages.

Nasal polyps

A patient comes in after just getting over a flare up of allergic rhinitis because she "can't smell anything" and works as a sommelier. Medical Hx reveals asthma and an allergy to ASA. Direct visualization reveals a pale, boggy mass on nasal mucosa and violet-colored turbinates. Eye exam shows cobblestoning of conjunctiva. What is it? What is first line Tx? What other test should be done in kids?

Nasal polyps intranasal glucocorticoids sweat chloride test to check for CF in kids

A patient comes in complaining of bad breath, fever, malaise and bleeding of the gums. On exam you note a grayish membrane that forms over the inflamed gingival margins, edema and ulceration of the gingiva. You also note cervical lymphadenopathy. Dx: Tx:

Necrotizing ulcerative gingivitis -Dental referral -2% viscous lidocaine -PO PNC Remove underlying factors

a 33- year-old complaining of right eye pain and irritation. He states that he wasn't wearing glasses, and while trimming his driveway with his weed trimmer, "something flew into my eye." Visual acuity is 20/20. Pupils are equal, round, reactive to light, and accommodation. Extraocular movements are intact. On physical examination, you note a tearing, red, and severely painful eye. DX: TX:

Ocular Foreign Body -Topical anesthesia -Irrigate -Attempt removal -Refer it not successful on 1st pass

A 10 year old boy is brought in by his father after c/o feeling like "something is in his eye" after playing in a sandbox. He is tearing a lot and his eyelid is spasming staining reveals an ice rink linear abrasion. When you shine a light in his eye to check it out, he shrinks back in pain. He does not wear contacts. What is it? what is the treatment?

Ocular foreign body/abrasion top erythro ung, Poly-T, sulfacetamide Remove: sterile irrigation/swab Small abrasions: don't patch Large: patch >5mm for <24 hrs Consult in 24 hrs Rust ring: remove at 24 hrs w/ rotating burr

a 47-year-old African American male presents for an ophthalmic examination. Medical history is significant for hypertension and type II diabetes mellitus. On slit-lamp examination, there is cupping of the optic disc, with a cup-to-disc ratio > 0.6. Tonometry reveals intraocular pressure of 45 mmHg (normal is 8-21 mmHg). Peripheral field vision loss is noted on visual field exam. What is the DX:

Open angle glaucoma

A 50-year-old man presents for a routine eye examination with no symptoms. He has elevated intraocular pressure of 25 mmHg in the right eye and 30 mmHg in the left eye. On dilated examination, the cup-to-disk ratio is 0.5 in the right eye and 0.8 in the left eye. Corneal thickness and gonioscopy are normal. Subsequent automated testing of visual fields demonstrates peripheral visual field loss greater in the left eye than in the right. Repeated automated visual field testing shows that the visual field defects are reproducible.

Open-angle glaucoma

a 47-year-old school teacher with a cough, hemoptysis, fever, chills, and weight loss that has persisted since he returned from a summer trip to China. A chest radiograph is concerning for infection and a sputum culture is positive for acid-fast organisms. Treatment for this patient's condition is begun. Three weeks later the patient returns to the clinic with decreased visual acuity for one day in his right eye. He also reports pain in the eye with movement but no other symptoms. The patient has a family history of glaucoma, diabetes mellitus, factor V Leiden and stroke. On physical examination, when a penlight is shined into the affected eye there is no pupillary constriction in either eye. what is the appropriate dx:

Optic Neuritis

A 30-year-old woman presents to the emergency room complaining of decreased vision in her right eye. The decreased vision started suddenly with blurred vision ("seeing as if through fog") and deteriorated over 2 days to the point that she was unable to see at all. She describes retro-ocular pain and pain with any movements of her right eye. On examination there is a right relative afferent pupillary defect, while the right optic disk appears normal. Her visual acuity is 20/200 as recorded on the Snellen chart in her right eye, with a large central visual field defect, and 20/20 in her left eye. She reports no prior history of ophthalmologic or neurologic problems. The remainder of the ophthalmologic and neurologic examination is normal.

Optic neuritis

A 47-year-old woman with a neurologic history of transverse myelitis 1 year ago and with residual spastic paraparesis presents with a 1-week history of acute loss of vision in both eyes. The decrease in vision started suddenly with blurred vision, and it has deteriorated over 2 days to the point that she is virtually unable to see at all with the right eye and barely able to distinguish faces with the left eye. There is an associated retro-ocular pain in both eyes and with eye movements. On exam there is a right relative afferent pupillary defect, and both optic disks appear slightly edematous. Her visual acuity is hand movements in the right eye and 20/200 in her left eye, with large bilateral central scotomata. The remainder of the ophthalmologic exam is normal. Neurologic exam reveals bilateral leg weakness and sensory loss with strength 3/5 in both legs on the MRC scale and a spastic ataxic gait.

Optic neuritis

characterized by painful visual loss and a swollen optic disc.

Optic neuritis

A 43 year old male patient with HIV comes in complaining of a cotton-like feeling in his mouth and loss of taste. Eating and swallowing hurt him as well. You try to take a scraping of his mouth and you notice a white, curd-like plaque on the buccal mucosa that EASILY scrapes off. What is this? What will KOH reveal? First line Tx?

Oral candidiasis budding yeast and pseudohyphae topical nystatin swish and swallow

A patient comes in with a painless, hairy looking plaque along the lateral tongue borders of his mouth. You try to scrape it, but no dice. Hx reveals HIV. What is it? What is the Tx?

Oral hairy leukoplakia Tx HIV this is self limited

a 17-year-old female complaining of a painful rash on her cheek. She says that it has come and gone a few times before and that she usually can feel itching and a tingling discomfort before a break out of the lesions. On physical exam, you observe clusters of small, tense vesicles on an erythematous base. DX? TX?

Oral herpes simplex PO acyclovir

A woman comes into the ER and looks distraught. Bruises are visible up and down her arms and her BF is acting really weird, trying to answer questions for her. She cant see well out of her left eye and PE reveals eye pain to constriction to bright light. Blood is visible on split lamp. What is the first image you order for Hyphema? other tests? What do you suspect? What is the tx?

Orbital CT w/o contrast to r/o open globe Call eye doc Blood panel/clotting factors suspect abuse muscle relaxants: rocuronium Eye shield Elevate head 30 degrees Bed rest

A 9-year-old boy is brought to the ER with redness and swelling around his eye that has been present for 1 day. His left eyelid is red, tender to touch, and swollen. image It will not open fully and he has a slightly decreased confrontational visual field in the left eye superiorly. He is afebrile and vital signs are normal. He denies any decrease in vision or double vision and his examination is significant for best corrected vision of 20/25 (right eye) and 20/25 (left eye). He has full motility of both eyes, has no afferent pupillary defect, and has eye pressures of 16 mmHg (right eye) and 18 mmHg (left eye). His conjunctiva and sclera are within healthy limits and the anterior chamber is deep and quiet. Fundus findings are normal in the left eye and the rest of his examination is within healthy limits. No masses are palpable. CT of orbits and sinus revealed absence of orbital fat stranding and subperiosteal abscess. Patient had opacified ethmoid and frontoethmoidal recess on the left side.

Orbital and Periorbital cellulitis

A child has fallen at play on to the corner of a table and has bruising around the eye. Since the event the child has been nauseated, may have vomited, and appears ill. On exam, the affected eye does not show any subconjunctival hemorrhage. The child is bradycardic and hypotensive. There is limitation of upgaze movement of the affected eye on upgaze, a maneuver that the child was resistant to perform.

Orbital fracture

A young man playing football received an elbow blow to the eye. He has double vision, and the eye looks sunken. On exam, he has periorbital ecchymosis and mild subconjunctival hemorrhage with a posterior limit. The affected eye is enophthalmic and hypoglobic. He has diplopia on upward gaze. Hess charting confirmed depression of upgaze movement with compensatory movement contralaterally.

Orbital fracture

A 20 year old female on her college swim team comes in with an itchy ear that feels "full of pressure." She complains that it is hard to hear and "gross stuff" is leaking out of her ear. Otoscopy reveals a red/edematous EAC. The tragus is painful when pulled What is it? What are the bugs? What is the treatment?

Otitis externa MC pseudomonas SA/E, GABHS, aspergillus, anaerobes, fungi dry: otic etoh/acetic acid remove debris abx: cipro + dexamethasone or AG + hydrocortisone drops

C Normal pneumatic otoscopy In contrast to acute otitis media, the tympanic membrane moves normally with pneumatic otoscopy in a patient with otitis externa.

Otitis externa can be differentiated from uncomplicated otitis media by which of the following physical examination findings? A Erythematous tympanic membrane B Mastoid tenderness C Normal pneumatic otoscopy D Posterior auricular adenopathy

An overweight man comes in after having a bad HA and N/V. PE reveals very high blood pressure. You check his eyes and notice a swollen optic disc. His vision is normal. What is this?

Papilledema

Idiopathic Sudden Sensorineural Hearing Loss

Patient presents with sudden, unilateral hearing loss. Physical exam show an abnormal Weber-Rinne test suggestive of sensorineural hearing loss. Patient recently recovered from COVID-19.

Mastoiditis

Patient will present as → a 10-year-old boy with otalgia, worsening over the last 5-days and associated with nasal congestion. The patient is afebrile with a temperature of 98 ° F. Examination reveals edema of the external auditory canal producing an anterior and inferior displacement of the auricle with percussion tenderness posteriorly.

External Ear Hematoma

Patient will present as → a 17-year-old on the high school varsity wrestling team was injured during a match. On physical exam, you note a fluctuant, tender edematous lesion of the anterior-superior outer portion of the right pinna.

Benign Positional Vertigo BPPV

Patient will present as → a 27-year-old male with intense nausea and vomiting that began yesterday. He states that he ran a 15-kilometer race in the morning and felt well while resting in a hammock afterward. However, when he rose from the hammock, he experienced two episodes of emesis accompanied by a sensation that the world was spinning around him. This lasted about one minute and self-resolved. He denies tinnitus or hearing changes, but he notes that he still feels slightly imbalanced. He has a past medical history of migraines, but he typically does not have nausea or vomiting with the headaches. Vitals are within normal limits. When the patient's head is turned to the right side and lowered quickly to the supine position, he claims that he feels "dizzy and nauseous." Nystagmus is noted in both eyes

Otitis media

Patient will present as → a 3-year-old previously healthy male is brought to your office by her mother. The mother reports the child has been crying and pulling at her right ear over the past 2 days and reports the patient has been febrile the past 24 hours. The patient's past medical history is unremarkable, although the mother reports the patient had a "common cold" a week ago which resolved without intervention. His temperature is 101.6 F, blood pressure is 100/70 mmHg, pulse is 120/min, and respirations are 22/min. The otoscopic exam is seen here.

Meniere's Disease

Patient will present as → a 32-year-old male with complaints of recurrent, episodic vertigo lasting up to 8 hours per episode for the past 3 months. The attacks generally last less than half an hour and are associated with decreased low-frequency hearing in the left ear along with nonpulsatile tinnitus in the ipsilateral ear. You obtain an audiogram which shows a low-frequency hearing loss in the left ear only.

Otitis Externa

Patient will present as → a 4-year-old girl who is brought to the clinic by her mother who states that the child has been complaining of progressively worsening ear pain and itchiness over the past week. Examination reveals left tragal tenderness and an edematous and closed canal. Weber lateralizes to the left.

Presbycusis

Patient will present as → a 72-year-old male with a progressively worsening hearing loss. He states that his trouble with hearing began approximately 7-8 years ago. He is able to hear when someone is speaking to him; however, he has difficulty understanding what is being said, especially when there is background noise. In addition to his current symptoms, he reports a steady ringing in both ears, and at times experiences dizziness. On physical exam, when a tuning fork is placed in the middle of the patient's forehead, the sound is appreciated equally on both ears. When a tuning fork is placed by the external auditory canal and subsequently on the mastoid process, air conduction is greater than bone conduction.

Tympanic membrane perforation

Patient will present as → a 9-month-old female with nasal congestion and cough is brought to your clinic by her mother who reports that the child is very fussy, has been tugging at her right ear, and refuses to eat. On physical exam, you note copious green/yellow nasal discharge and right-sided otorrhea. An otoscopic exam reveals a significant amount of clear/white discharge obstructing your view. With careful examination, you are able to observe a ruptured right tympanic membrane.

Cholesteatoma

Patient will present as→ a 43-year-old male with a "lifelong" history of chronic ear infections and episodic purulent drainage from his right ear canal. The patient currently is without symptoms. Examination of the ear shows a clear external canal, but the tympanic membrane is retracted and there is a pocket of white material and an opacity of the pars flaccida. The Weber test lateralizes to the right and Rinne shows air conduction > bone conduction on the left and bone conduction > air conduction on the right.

a 19-year-old male who you are seeing for follow-up from the urgent care where he was seen 2 days earlier with a sore throat. The patient is febrile (102°F), has a muffled (hot potato) voice, and extreme difficulty opening his mouth (trismus). He opens it just far enough for you to note uvular deviation. Dx: Tx:

Peritonsillar Abscess - IV hydration (pedi) -Surgery (no relief) -Consult ENT -Admit if : drooling, dyspnea, trismus, severe pain -I&D IM/ IV ampicillin or clindamycin then PO amoxicillin, augmenting or clindamycin -Steroids -Tonsillectomy

A 25 year old patient comes in with a "muffled hot potato voice." He has been having trouble swallowing and his jaw has been locking up. PE reveals severe UL pharyngitis and a high fever. + for anterior LAD What is it? bugs? What test can you order? Tx?

Peritonsillar Abscess (Quinsy) poly: S. pyogenes, SA, resp stuff CT I&D w/ needle asp augmentin or clindamycin Tonsillectomy if need.

An obese, 71 year old man with high blood pressure comes in with a gushing bloody nose out of BOTH nares. He ran out of his BP medication 2 days ago and is running quite high. Head trauma has been ruled out and the patient is free from any bleeding disorders What is it? MC site? MC first treatment? What else should you do?

Posterior Epistaxis sphenopalatine artery branches & Woodruffs plexus Balloon catheter Tx high BP

Vocal fold polyps conservative tx unless large

Professor presents for voice concerns. You see the following.

Treatment of Open angle glaucoma

Promote Drainage: -Prostaglandin (latanoPROST) -Cholinergic (pilocarpine, carnacol) -Epinephrine (Dipivefrin) Decrease Production -Betablockers (timOLOL) -Carbonic Anhydrase (DorzolAMIDE) Both -Alpha agonist (Brimonidine)

Septal perforation Symptomatic Sx

Pt 7 days s/p septoplasty presents for surgical f/u w symptoms of whistling, bleeding and crusting. Able to see through the septum on PE.

Fibromas Sx

Pt present for the following. The site is painless. She states she has a bad habit of biting her inner cheek.

Sialolithiasis (Ca stone blocking salivary gland) Stone removal (manual/ probe/ sx)

Pt present sfor pain and local swelling after eating meals. PMHx of 2 episodes of bacterial infections of the parotid gland.

Vocal cord paralysis BL = surgical emergency

Pt presents 4 day s/p thyroidectomy for difficulty breathing. PE + stridor w deep inspiration

Hyphema Referral, bed rest, elevated head of the bed, decrease IOP w I diuretics

Pt presents after MVA for UL vision loss and pain.

Labyrinthitis Self limiting Diazepam

Pt presents for 3 days of acute onset of continuous and severe vertigo. Reports UL hearing loss and ringing in the ear. No PMHx or FHx. PE +Romberg test, all other findings are normal. Pt reports having a viral URI 2 wks ago.

BPPV Dix Hallpike Meclizine/ Diazepam Eppley maneuver

Pt presents for 3 wks h/o sensation of spinning/ moving for about 30 seconds at a time. Happens often, most commonly after turning her head quickly. No hearing loss, ringing in the ears or loss of balance. No PMHx or FHx.

Presbyacusis Hearing aids

Pt presents for BL progressive hearing loss of high frequency sounds. Has difficulty hearing words at restaurants and concerts. Used to work in construction. FHx of dad w similar symptoms. No PMHx.

Ulcerative blepharitis Eyelid hygiene, topical abx

Pt presents for BL red rimmed eyelids w scales around the base of lashes

Herpetic Simplex Keratitis Fluorescein stains + blue light --> branching/ dendritic lesion Topical antivirals + referral to ophth ASAP

Pt presents for UL pain, tears and FBS.

Oral erosive lichen planus Topical stds.

Pt presents for annual physical. PE + ulcerations with "surrounding border of white lines". Reports frequent use of advil for HAs.

Meniere's disease Low sodium diet Diuretics Oral antihistamines

Pt presents for chronic episodic vertigo lasting about 2 hours at a time. Reports UL low frequency hearing loss. PE +sensorineural hearing loss on weber rinne and nystagmus. No PMHx or FHx.

Acoustic neuroma MRI w contrast Sx

Pt presents for gradually worsening UL hearing loss and feeling of loss of balance. Reports ringing in the ear and facial numbness. Neuro PE + for deficit in finger rub test.

Laryngitis Self limiting

Pt presents for hoariness and loss of voice. Feverish on exam. Rest of PE is normal. All labs and cultures are wnl. No PMHx.

Retropharyngeal abscess CT I+D, IV abx, admit to ICU

Pt presents for neck stiffness and pain. No PMHx, though pt was tx'd w IV clindamycin for peritonsillar abscess 2 wks ago.

Corneal ulceration (pseudomonas) Fluorescein stain -> round ulcers w dense corneal infiltrate Referral to ophtho ASAP + abx

Pt presents for pain and photophobia. No trauma or PMHx. Pt does wear contact lens.

Bacterial Keratitis Fluorescein stain --> round white spot w ulcer Topical abx (fluoroquinolone)

Pt presents for pain and photophobia. PE + for hypopyon. No PMHx. Pt is a contact lens wearer.

bacterial rhinosinusitis self limiting, supportive care severe = augmentin

Pt presents for purulent green nasal d/c and facial pressure over maxillary sinus.

Gingivitis Tx underlying

Pt presents for red, painful and easily bleeding gums. PMHx of epilepsy controlled w anticonvulsants and GAD tx'd w escitalopram.

Allergic rhinitis Dx = high serum IgE Flonase (intranasal corticosteroid sprays) Antihistamine

Pt presents for runny nose, congestion, sneezing and itchy eyes. PE + for allergic shiner and nasal salute.

Acanthamoeba keratitis Ophtho referral ASAP Clotrimazole long term

Pt presents for severe pain in the R eye and the following on confocal microscopy. No PMHx, though pt does wear contacts. They clean contacts in tap water.

Seborrheic Blepharitis Eyelid hygiene, topical abx

Pt presents for shiny lid margin w telangiectasis + greasy scales

Peritonsillar abscess Needle aspiration/ I+D IV Clindamycin

Pt presents for sore throat, UL pain w swallowing and hot potato voice.

Diphtheria Removal of membrane Antitoxin Abx

Pt presents for sore throat, hoarseness and "bull neck". PE reveals the following.

tympanic membrane perforation Keep ears clean and dry Abx drops (ofloxacin)

Pt presents for sudden L ear pain and hearing loss.

Sialadenitis US/ CT IV abx + increase salivary flow w hydration and warm compresses

Pt presents for the following + increased pain after meals.

Traumatic ulcer Remove irritating source Palliative care

Pt presents for the following removable lesion. Pt wears dentures.

Dacryocystitis (staph) Systemic abx (augmentin) for acute + warm compresses

Pt presents for the following. No PMHx.

dacryoadenitis Warm compresses, NSAIDs

Pt presents for the following. No PMHx.

Orbital cellulitis CT Referral to ophth ASAP Immediate IV broad spectrum abx

Pt presents for the following. PE + for decreased EOM.

Herpes Zoster Ophthalmicus Consult Optho ASAP! Systemic antiviral

Pt presents for the following. Sxs were proceeded by fever and HA. PE+ for increased IOP and periorbital edema BL.

Fungal keratitis Topical antifungals +/- abx

Pt presents to EAC for the following. Pt is visiting the US from Costa Rica. PE + for grey/ white infiltrate and dry w feathery borders and hypopyon.

Septal hematoma Sx w/n 3-5 days to remove clot

Pt presents to ED after being hit in the face for concerns for concussion. PE reveals the following.

Vocal fold cyst

Pt presents to ENT for hoarseness. Sxs fluctuate from week to week.

Chalazion Warm compresses

Pt presents w the following. Has slowly developed over the past week. Painless mass.

Blow out fracture CT = teardrop sign Referral ASAP

Pt presents with the following appearance. She is unable to look up and the effected eye is sunken back in the socket.

Retinal artery occlusion Referral ASAP Decrease IOP

Pt w PMHx of AFIB, CAD and one TIA presents for sudden UL vision loss. PE + retinal swelling w cherry red spot on the fovea and cotton wool spots.

Fungal Rhinosinusitis Emergent sx IV antifungals (Amphotericin B)

Pt w PMHx of COPD controlled w long term use of corticosteroids presents for severe facial pain and straw colored nasal d/c. PE + for tissue necrosis on nasal turbinate

Erythroplakia Bx Complete excision

Pt w PMHx of ETOG abuse presents for the following well demarcated plaque. Spot does not rub off and is not painful.

Oral reticular lichen planus

Pt w PMHx of GAD presents for annual physical. PE + for lacy web-like, white threads that are slightly raised.

Allergic Conjunctivitis Topical antihistamines Artificial tears

Pt w PMHx of allergic rhinitis presents for itching, tearing and milky d/c from both eyes.

Nasal polyps Intranasal corticosteroids Surgical removal if large

Pt w PMHx of asthma, ASA allergy and CF presents for a WCC. The following is noted on PE.

Olfactory dysfunction Smell identification test Endoscopic sinus sx

Pt w PMHx of nasal polyps and asthma presents for altered taste. Labs and CT are all normal.

rhinitis medicamentosa stop afrin

Pt w PMHx of seasonal allergies controlled w long term daily use of topical Afrin presents for rebound nasal congestion.

Oropharyngeal cancer

Pt w PMHx of tobacco and ETOH dependence presents for UL, neck mass and wt loss

Leukoplakia Bx Complete excision (MC oral precx)

Pt w PMHx of tobacco and ETOH dependence presents for the following spot that does not rub off.

Periodontitis Professional cleaning and sx Systemic abx

Pt w PMHx of tobacco dependence and DM presents for teeth mobility. PE + for recessed gingival margins.

Oral SCC

Pt w PMHx of tobacco dependence presents for the following raised, firm and painful lesion.

Aphthous ulcers Self limiting; topical corticosteroids or viscous lidocaine 2-5% after meals

Pt w PMHx of uncontrolled GAD presents for painful ulceration surrounded by red halos.

Vitreous hemorrhage Referral ASAP

Pt w h/o DM and RA presents for sudden onset of vision loss + floaters. The following is seen on ophthalmoscope exam.

Posterior epistaxis Emergent care Posterior nasal packing Ligation of posterior nasal artery supply

Pt w h/o uncontrolled HTN and CAD presents for a bleed at the palatine artery

Viral conjunctivitis Cool compresses Hyguene

Pt w no PMHx presents for the following + clear, watery d/c. PE + for preaurciular LAD.

Bacterial conjunctivitis (s aureus) Swab/ culture topical abx Ophtho emergency if gonococcal

Pt w no PMHx presents for the following. + Purulent d/c and redness. Sxs are worse in the morning, eyes feel like they are glued shut.

Viral rhinosinusitis Self limiting

Pt w no PMHx presents w nasal congestion, clear and watery rhinorrhea, sneezing and HA. Nasal swab and throat swab negative.

A 35-year-old woman of Middle-Eastern origin presents with a lesion at the corner of her eye by her nose. This has been present for several years. She occasionally gets ocular irritation but is worried about the cosmetic appearance. A raised fleshy lump can be seen at the nasal aspect of her cornea, which encroaches about 2 mm onto the cornea. The eye otherwise looks normal with vision recorded at 20/16.

Pterygium

A 45-year-old man presents with ocular irritation, burning, and tearing with the presence of a white/red lesion on the side of his eye by his nose, which he reports has been growing toward the pupil for at least 5 to 10 years. He has been residing in the US for the past 5 years but previously lived in West Africa. He reports slight blurring of his vision. He has a triangular-shaped piece of fleshy, fibrovascular tissue encroaching onto the nasal aspect of his cornea by about 3 to 4 mm. At its apex it has a whitish head, and it is continuous with the conjunctiva at its base, which is about 6 mm in length. The eye otherwise looks normal with a best spectacle-corrected vision of 20/20.

Pterygium

A man comes into the clinic c/o a triangular mass on the front of his eye ball. He's here visiting, but normally lives in Arizona. His left eye is very irritated and it's affecting his sight. What is it and what is the treatment at this point?

Pterygium removal - it's too bad to watch/use fake tears

fleshy triangle shaped protrusion on the inner bulbar conjunctiva

Pterygium is a complication of exposure to ultraviolet light and wind. It consists of hyaline and elastin tissue. If it encroaches on the cornea, surgical removal is indicated.

Vocal fold nodules ENT referral voice therapy and rest

Public speaker presents for annual physical PE + coarse, breathy voice.

Herpes Zoster Ophthalmicus

Reactivation of​ VZV in ​ophthalmic division​ of ​trigeminal ganglion

Patient will present as → a 74-year-old man presents with sudden vision loss in his right eye. He has a medical history of hypertension and coronary artery disease and new onset atrial fibrillation. On physical exam, a carotid bruit is auscultated. His visual acuity is light perception. Confrontational visual fields reveal a dense scotoma, and a penlight examination shows an afferent pupillary defect. Dilated funduscopic examination shows retinal whitening with a cherry-red spot in the fovea. DX:

Retinal Artery Occlusion

Patient will present as → a 65-year-old man complaining of a sudden unilateral vision loss which he describes as "a curtain or dark cloud lowering over my eye." This was preceded by small moving flashing lights, and floaters. The fundoscopic exam reveals a detached superior retina. Dx:

Retinal Detachment

A 67-year-old man presents with a 2-day history of sudden visual loss in his right eye. He is slightly myopic and had successful cataract extraction with intraocular lens implantation 3 years earlier. He does not remember this eye ever having been injured. No pain was associated with the vision loss, and his blood pressure is normal with medication. The patient describes the loss of vision as a veil covering the visual field.

Retinal detachment

An old man comes into the ER complaining that he saw flashes, floaters and felt like a curtain came down slowly over his left eye. He is in no pain. On physical exam, you notice that the vitreous humor is flapping and that there seems to be a clumping of brown pigment in it. What is it? What is the treatment?

Retinal detachment Keep supine, get consult Keep head turned toward detachment Laser cryo or ocular surgery

A 65-year-old man with a history of hypertension and hypercholesterolemia notices sudden, painless vision loss in his right eye. The vision loss is limited to the superonasal quadrant of his visual field. He first noticed the visual field loss approximately 6 weeks ago. For the past 2 weeks, however, he has started to have blurred vision centrally, making reading difficult.

Retinal vein occlusion

Patient presents with chronic nasal congestion. She is very frustrated as she has been treating it with Afrin for months with no resolve.

Rhinitis medicamentosa

An 80 year old woman comes in with a Hx of rheumatoid arthritis. She c/o a tender, purplish eye and blurry vision. Her eye is red and she describes the pain as dull, but severe. Slit lamp shows profound dilation of the deep episcleral-vascular plexus. What is it? What is the treatment?

Scleritis indomethacin

a 42-year-old white female complaining of a severely painful right eye. The pain is a constant, boring pain that worsens at night and in the early morning hours and radiates to the face and periorbital region. Additionally, she reports a headache, watering of the eye and ocular redness. DX TX

Scleritis -Refer -Systemic therapy w/ NSAIDS, glucocosteroids or other combo

A 28 year old male boxer comes into the emergency room after taking a punch to the face. He has swelling, tenderness, crepitus and a deformity of the nose. Patient states that they had come epistaxis, and rhinorrhea. What must you rule out? and if seen what is the next step?

Septal Hematoma Immediate ENT consult

Patient presents with whistling, crusting, and bleeding from their nose. The patient confirms illicit drug use. What do you suspect?

Septal perforation

A 15 year old male comes in complaining of "feeling under water". On exam of the R ear you note clear fluid with bubbles behind the TM, decrease TM mobility, and boney landmarks are intact, but dull. While preforming a Webber test, sound lateralizes to the R ear. On Rhine test of the R ear BC>AC. Dx? Tx?

Serous Otitis Media -Refer to ENT (>8 wks after tx) -Oral/topical decongestions -Autoinsuffilation -TM tubes -IN corticosteroids -Careful it can lead to AOM

a 34-year-old previously healthy male with complaints of facial pressure and rhinorrhea for the past 3 weeks. The patient reports that several weeks prior, he had a "common cold" which resolved. However, he has since developed worsening facial pressure, especially over his cheeks and forehead. He reports over 1 week of green-tinged rhinorrhea. His temperature is 100.1 F (37.8 C), blood pressure is 120/70 mmHg, pulse is 85/min, and respirations are 15/min. The nasal exam reveals edematous turbinates and purulent discharge. The patient has facial tenderness with palpation over the involved sinus. Dx: Tx:

Sinusitis -PO Abx 7-14 days -Augmentin (severe) -Saline Flush

Symptoms of cataract

Slow progressive vision loss/ blurriness -Glare -Fixed spots -reduced color perception -Brow to yellow disocoloration

A 34-year-old man complains of double vision after recovering from facial trauma that included a fracture of the left orbital floor. He describes the double vision as manifesting with one image on top of the other. The patient complains that he is unable to function unless he closes one eye. Exam shows hypotropia of the left eye, which becomes larger when the patient tries to look up. The remainder of the ocular exam is normal. A CT scan of the orbits demonstrates orbital tissue herniating through a fracture in the floor of the left orbit, and the patient is diagnosed with restrictive incomitant left hypotropia.

Strabismus

A 4-month-old healthy girl with normal prenatal and birth history is brought in by her parents, who note that both her eyes are looking toward the nose. Exam shows a large-angle esotropia with freely alternating fixation (i.e., each eye fixates objects, with no preference for either eye). No significant refractive error is present, and the remainder of the eye exam is normal. The infant is diagnosed with infantile esotropia.

Strabismus

A 3-year-old girl brought to you by her mother who is worried about her daughter's "lazy eye." She reports that her daughter's symptoms are exaggerated when she has a cold. Past medical history is negative for trauma or headaches. The patient has an asymmetric corneal light reflex and the cover/uncover test reveals a right-sided esotropia. You refer the patient to a pediatric ophthalmologist. DX: TX:

Strabismus -glasses

A 27 y/o with a hx of HTN male comes into the office for a red eye. The patient stated that he looked into the mirror this morning and his whole eye turned a dark red color. He denies any pain or vision changes. On exam you note dense blood in the subconjunctival space. DX TX

Subconjunctival Hemorrhage -Reassurance 2-3 weeks to clear -Artificial tears -refer if suspect something bad

How would you treat a patient with viral sinusitis?

Symptomatic Saline Irrigation Nasal steroids Antihistamines Return if not resolved in 1 week to give abx because it's probably bacterial by then.

How would you treat a patient with viral rhinitis?

Symptomatic Decongestants Nasal saline irrigation

How would you treat a patient with bacterial sinusitis?

Symptomatic for first 7 days (Saline irrigation, nasal steroids, antihistamines) If no improvement, Amoxicillin

a 9-month-old female with nasal congestion and cough is brought to your clinic by her mother who reports that the child is very fussy, has been tugging at her right ear, and refuses to eat. On physical exam, you note copious green/yellow nasal discharge and right-sided otorrhea. An otoscopic exam reveals a significant amount of clear/white discharge obstructing your view. With careful examination, you are able to observe a ruptured right tympanic membrane. DX: Tx:

TM Perforation - May heal spontaneously -Tx OM w/ Abx (floxin) + dexamethasone -Avoid water & aminoglycosides

A 21 year old woman comes into your office after spending all weekend working backstage at a music festival. Her left ear started "hurting so badly out of nowhere" and she was having trouble hearing. This morning, her ear felt better, but she noticed blood on her pillow when she woke up. Otoscopy would reveal what? What is the treatment? what test should be avoided?

TM perforation heal on own +/- ofloxacin avoid pneumatic otoscopy

A 1-day-old infant being examined in the newborn nursery is noted to have a central, 4 mm cataract affecting his right eye. Which of the following is the most appropriate management for this patient? A. Cataract surgery within the next 6 weeks B. Cataract surgery within the next year C. Observation every 3 months D. Observation every 6 months

The answer is A. EXPLANATION: Congenital cataracts that are large and affect visual acuity (e.g., central) must be surgically corrected within the first two months of life (A) to avoid the development of deprivation amblyopia. Observation (C, D, and E) or delayed surgery (B) may result in permanent deprivation amblyopia.

A patient presents with epistaxis from the right nares, along with direct pressure to the nares and elevation of the head. Which of the following is an appropriate initial treatment? A. Phenylephrine spray and anterior packing B. Triamcinolone spray and anterior packing C. Azelastine spray and anterior packing D. Momentasone spray and anterior packing

The answer is A. EXPLANATION: Phenylephrine is a topical decongestant, and acts as a vasoconstrictor to aid in stopping minor anterior septal epistaxis. Triamcinolone and momentasone are nasal steroids used for allergic rhinitis. Cromolyn sodium is a mast cell stabilizer, and azelastine is a topical H1 selective antihistamine, used for allergic rhinitis.

A 23-year-old male presents to the clinic complaining of left anterior neck pain that developed over the past week following recovery from an acute upper respiratory infection. On physical exam a tender mass is felt anterior to the left sternocleidomastoid muscle from the mandible inferiorly to the level of the cricoid cartilage. Which of the following is the most likely diagnosis? A. Branchial cleft cyst B. Dermoid cyst C. Peritonsillar abcess D. Salivary gland tumor

The answer is A. EXPLANATION: The development of a neck mass in a young adult following URI is consistent with branchial cleft cyst (A) and thyroglossal duct cyst (E). The location of this mass away from the midline and anterior to the SCM is most consistent with branchial cleft cyst (A). The location of the mass and history are inconsistent with dermoid cysts (B), which are typically midline, peritonsillar abcesses (C), which would be located in the retropharyngeal space, and salivary gland tumors (D), which would be located in the parotid, submandibular, or submental salivary regions.

A 54-year-old female presents complaining of decreased visual acuity to her right eye over the past few hours. She denies pain, and describes having wavy vision and seeing flashes of light. Her visual acuity in the affected eye is 20/200. What condition best describes the following physical finding? A. Retinal detachment B. Central retinal artery occlusion C. Open angle glaucoma D. Angle closure glaucoma E. Optic neuritis

The answer is A. EXPLANATION: The image demonstrates a detached retina. The superior aspect of the retina appears wavy and flowing.

A 43-year-old female presents to the outpatient clinic complaining of itching and irritation of her right eye. She denies decreased vision or photophobia. On physical exam the patient's eye has the following appearance: Which of the following is the most appropriate management for this patient's condition? A. Daily lid cleansing and application of bacitracin ophthalmic ointment 500 units/g B. Doxycycline 100 mg by mouth once daily C. Incision and drainage D. Referral to an ophthalmologist

The answer is A. EXPLANATION: The patient's symptoms of itching and irritation of the lid margin with an inflamed eyelid and eyelash scaling is consistent with mild anterior blepharitis, which is initially treated with cleansing and the potential addition of a topical antistaphlococcal antibiotic (A). Answers (C) and (E) are appropriate treatments for a hordeolum. Doxycycline once daily (B) can be used as a long-term treatment for posterior blepharitis. The condition doesn't warrant referral (D).

As a diver descends for a deep water dive, at about 10 feet of depth he begins to feel nausea, severe ear pain, and develops vertigo and vomiting. What is the most likely cause of his symptoms? A. Decompression sickness B. Decreasing pressure in the middle ear C. Benign paroxysmal positional vertigo D. Increasing pressure in the middle ear E. Equalization of pressure between the middle ear and eustachian tube

The answer is B. EXPLANATION: Boyle's law states that as a diver descends, the increasing external pressure causes an equal decrease in pressure in the middle ear, which must be equalized during the descent. If the middle ear pressure is not equalized, the tympanic membrane becomes severely retracted, due to the negative middle ear pressure. This can result in hemotympanum, hemorrhage, or tympanic membrane perforation. Ascent causes increased pressure in the middle ear as the external pressure is decreased. Equalization techniques must also be used to prevent a tympanic membrane perforation. Decompression sickness occurs on ascent, when nitrogen gas bubbles are forced into the middle ear, and vascular and lymphatic spaces.

A 60-year-old male presents with complaints of irritation and a white plaque on his tongue. He denies pain. During physical exam you are unable to remove the white plaque from the mucosa with a tongue depressor. What is the most likely diagnosis, represented as follows? A. Oral thrush B. Leukoplakia C. Geographic tongue D. Glossitis E. Lichen planus

The answer is B. EXPLANATION: Oral leukoplakia cannot be removed from the mucosa using a tongue depressor like oral thrush can. Lichen planus can mimic candidiasis, squamous cell carcinoma, or hyperkeratosis, and requires a biopsy to diagnose. Glossitis is a generalized inflammation, and loss of papillae of the tongue is caused by vitamin deficiencies, medication reactions, auto immune reactions, or psoriasis. Geographic tongue is an asymptomatic serpiginous area of atrophy and erythema of the anterior tongue. The condition is self-limiting.

A 65-year-old male presents to you with complaints of decreasing hearing, along with difficulty discerning words when in conversations in noisy environments, such as restaurants. His only medication is simvastatin for hyperlipidemia. The following is his audiogram. He has bilateral decreased high frequency hearing loss, and decreased speech recognition. What is the most likely diagnosis? A. Vestibular schwannoma B. Presbycusis C. Presbystasis D. Cerumen impactions E. Vestibulobasilar insufficiency

The answer is B. EXPLANATION: Presbycusis is age related bilateral loss of high frequency hearing, and decreased word recognition. Presbystasis is age related balance disorder. Vestibular schwannoma (acoustic neuroma) causes unilateral hearing loss. Vestibulobasilar insufficiency results from atherosclerosis of the vertebral arteries, and can cause many symptoms including double vision, speech defects, vertigo, ataxia, and drop attacks.

A 45-year-old male presents with purulent discharge from his right ear for three weeks. He states that despite being treated by his family doctor for an ear infection one month ago, the problem continues to get worse. Upon exam, you note purulent discharge in the ear canal, an erythemic tympanic membrane, and a possible perforation. What are the pathogens most likely to culture positive? A. Strep pneumoniae B. Pseudomonas aeroginosa C. Escherichia coli D. Candida albicans E. Mycoplasma pheumoniae

The answer is B. EXPLANATION: The clinical vignette describes a chronic otitis media. Usually, this refers to a complication of acute otits media with perforation. Pathogens that culture from these infections are usually pseudomonas, proteus, or staphylococcus aureus. Strep pneumoniae is often seen in acute otitis media. E.coli is a urinary tract pathogen. Candida albicans is a cause of vaginitis, and mycoplasma is a respiratory pathogen.

A 43-year-old woman presents to the outpatient clinic complaining of right eye redness, photophobia, and pain. She notes some blurred vision and denies the presence of discharge. On physical exam her visual acuity is 20/20 left eye, and 20/60 right eye. Her right eye has circumcorneal injections and the pupil is 3 mm and responds poorly to light. Her left pupil is 5 mm and responds well. Fluorescein staining of the eye is unremarkable and intraocular pressures are normal. Which of the following treatment regimens should be prescribed? A. Homatropine 5% solution four times daily B. Homatropine 5% solution four times daily and prednisolone 1% solution every 1 or 2 hours while awake C. Prednisolone 1% solution every 1 or 2 hours while awake

The answer is B. EXPLANATION: The patient's presentation of acute uveitis is best treated with topical corticosteroids and cycloplegics (B) once infectious causes (e.g., HSV) have been ruled out. The addition of a cycloplegic helps reduce pain. Antibiotic drops (D) aren't indicated for acute uveitis.

A 37-year-old male presents to your office with a history of vision loss in his right eye. He denies any pain, and states that the vision loss occurred suddenly. He noted there was a wavy, "curtain-like" visual disturbance preceding the vision loss. Upon physical exam you notice a cherry red spot over the macula and retinal pallor. What is the most likely diagnosis? A. Macular degeneration B. Retinal detachment C. Central retinal artery occlusion D. Cerebrovascular accident

The answer is C. EXPLANATION: Central retinal artery occlusion is characterized by a sudden, painless vision loss. A cherry red spot is characteristic on the macula, along with pallor to the retina.

A 73-year-old female with type 2 diabetes, hypertension, and hyperlipidemia presents to the outpatient clinic complaining of left ear pain, and a yellowish-green, foul-smelling discharge that began about 3 weeks ago. On physical examination, the patient is afebrile and examination reveals a markedly edematous left ear canal draining purulent, green discharge. The tympanic membrane is unable to be visualized. Which of the following is the most likely causative agent for this patient's diagnosis? A. Escherichia coli B. Moraxella catarrhalis C. Pseudomonas aeruginosa D. Staphylococcus aureus

The answer is C. EXPLANATION: Malignant otitis externa is most commonly caused by pseudomonas (C). E coli (A) and S aureus (D) are less common causes of otitis externa, while S pneumoniae (E) and M catarrhalis (B) are common etiologies of acute otitis media.

Jane, a 21-year-old female, was seen in the office 10 days ago and was diagnosed with perennial allergic rhinitis and sent home with instructions for increased fluids, decongestants, and nasal steroids. She returns today with worsened symptoms of malaise, low-grade fever, nasal discharge, cough that is worse at night, mouth breathing, early morning unilateral pain over sinuses, and congestion. Physical examination reveals thick purulent nasal discharge, postnasal discharge visible in the posterior pharynx, periorbital swelling, and tenderness of sinuses upon palpation. She is 36-weeks pregnant and allergic to penicillin. Of the following, what is the most appropriate antibiotic? A. amoxicillin B. trimethoprim-sulfamethoxazole C. clindamycin D. levofloxacin

The answer is C. EXPLANATION: Most patients with a diagnosis of acute rhinosinusitis based on clinical grounds improve without antibiotic therapy. The preferred initial approach in patients with mild to moderate symptoms of short duration is therapy aimed at facilitating sinus drainage, such as oral and topical decongestants, nasal saline lavage, and—in patients with a history of chronic sinusitis or allergies—nasal glucocorticoids. Adult patients who do not improve after seven days, children who do not improve after 10 to 14 days, and patients with more severe symptoms (regardless of duration) should be treated with antibiotics. Empirical therapy should consist of the narrowest-spectrum agent active against the most common bacterial pathogens, including S. pneumoniae and H. influenzae—e.g., amoxicillin. But amoxicillin is contraindicated in patients with urticarial reactions to penicillins, and quinolones are similarly contraindicated in pregnancy. trimethoprim-sulfamethoxazole is contraindicated in the third trimester of pregnancy. The best choice is clindamycin.

A 55-year-old male presents with severe swelling to his left eye. He denies injury or allergies. He states that he has had a severe sinus infection for the past two weeks. What is the most likely diagnosis for the following physical finding? A. Allergic reaction B. Eyelid abcess C. Orbital cellulitis D. Erysipelas

The answer is C. EXPLANATION: The image shows a severe orbital cellulitis. These infections often spread from paranasal sinus infections. Multiple pathogens may be involved, such as s.aureus, s. pheumoniae, and anaerobes. An eyelid abscess would be more localized. Allergic reactions that cause angioedema around the eye are usually bilateral, and also pruritic.

An 18-year-old female presents with two weeks of severe sore throat and fatigue. Her exam shows an exudative tonsillitis. A mono-spot test is positive, and a rapid strep test is positive. Which of the following medications should be avoided? A. Erythromycin B. Clindamycin C. Cephalexin D. Ampicillin E. Prednisone

The answer is D. EXPLANATION: Ampicillin should be avoided, because a high percentage of mononucleosis patients develop a fine, non-allergic maculopapular rash when given ampicillin class drugs. The remaining antibiotics are appropriate for treating group A strep. Prednisone is used to reduce the pain and inflammation associated with severe tonsillitis.

A 20-year-old male presents with cough, nasal congestion, and a low grade fever for one week. His cough seems to be getting worse, which is the reason for his visit. His past medical history includes asthma and nasal polyps. On physical exam, his temperature is 101°F, his pharynx is erythemic, and there is grey nasal discharge with a few nasal polyps seen using a nasal speculum. His lungs have a few expiratory wheezes bilaterally. What medication is to be avoided in this patient? A. Penicillin B. Acetaminophen C. Erythomycin D. Aspirin E. Ciprofloxacin

The answer is D. EXPLANATION: Aspirin should be avoided in patients with asthma and nasal polyps. Aspirin can precipitate bronchospasm in these patients, due to immunologic salicylate sensitivity.

A 63-year-old female complains of a 5-day history of a persistent left-sided headache, which she has not experienced before. She also notes a tender swollen area around her left temple, which appeared around the same time. On examination you note tenderness and prominence of the left temporal artery. You order an ESR, which is 75 mm/h. What is your best course of action at this time? A. Repeat the ESR in 72 hours. B. Begin prednisone 20 mg/d and increase if symptoms persist. C. Refer to a rheumatologist for appointment next month, with a trial of nonsteroidal anti-inflammatory drugs (NSAIDs). D. Begin prednisone 60 mg/d immediately. E. Refer for a temporal artery biopsy next week, with a trial of hydrocodone for analgesia.

The answer is D. EXPLANATION: The correct answer is (D). The patient's history and physical examination findings point to giant cell arteritis (temporal arteritis) as the most likely cause, prompting immediate treatment with high-dose prednisone to prevent visual loss. The patient meets the criteria for clinical diagnosis of giant cell arteritis without a temporal artery biopsy, but it is recommended for definitive diagnosis due to the complications associated with long-term corticosteroid treatment. Treatment with prednisone should not be withheld while waiting for a temporal artery biopsy. NSAIDs and hydrocodone do not prevent the complications of temporal arteritis.

A 23-year-old man presents to the outpatient clinic for follow-up from a recent urgent care visit. He complains of sore throat, fever, fatigue, myalgias, and a rash that started 5 days ago, and have worsened since he was seen in the urgent care 3 days ago. The patient appears non-toxic with a temperature of 39.4 degrees Celsius. Physical exam reveals pharyngeal and tonsillar erythema without exudates, generalized lymphadenopathy, a morbilliform rash on his trunk, and no hepatosplenomegaly. A rapid strep screen and Monospot performed at the local urgent care were reportedly negative. Which of the following laboratory tests is most likely to confirm the expected diagnosis? A. Complete blood count B. Cytomegalovirus titer C. Group A beta-hemolytic strep culture D. HIV viral load

The answer is D. EXPLANATION: The patient presentation is consistent with acute retroviral syndrome, which is best confirmed during this initial presentation phase through direct testing for the HIV virus, such as an HIV viral load (D). The lack of tonsillar exudates, a negative monospot, and presence of generalized adenopathy make infectious mononucleosis (B and E) less likely. A CBC (A) may show lymphopenia and support the diagnosis, but it doesn't confirm the diagnosis.

A 3-year-old girl presents to the otolaryngologist for evaluation of a persistent left ear infection and drainage that have failed to respond to multiple antibiotic regimens. Which of the following is the most likely causative organism for this patient's condition? A. Aspergillus B. Chlamydia pneumoniae C. E. coli D. Streptococcus pneumoniae E. Staphylococcus aureus

The answer is E. EXPLANATION: Chronic otitis media is typically caused by P. aeruginosa, H. influenzae, S. aureus (D), Proteus species, Klebsiella pneumoniae, or Moraxella catarrhalis. Aspergillus (A) and E. coli (C) are associated with otitis externa and streptococcus pneumoniae (D) is the most common bacterial cause of otitis media. Reference:

An 18 month old female is brought to the pediatricians office with a history of cough, fever of 102, and decreased fluid intake. Her immunizations are not up to date as the family just moved to the United States from out of the country. On physical exam she is drooling and sitting up in a "tripod position" with mild stridor. What is the most appropriate treatment indicated for this condition? A. Humidified air B. Albuterol nebulizer C. Budesonide nebulizer D. Recemic epinephrine nebulizer E. Ipratropium nebulizer

The clinical presentation suggests epiglotitis. This is an emergent airway condition. The anesthesiologist , or the pediatric otolaryngologist must be called to stand by to intubate or insert a tracheostomy if the patients airway closes. Racemic epinephrine via nebulizer relieves much of the edema to the upper airway in a patient with epiglotitis. It is a stabilizing measure until definitive care can be arranged. Oxygen and antibiotics should administered emergently also. No x-rays are indicated when the presentation is classic. Albuterol is a beta-agonist used for treatment of asthma. Budesonide,a steroid and ipratropium, an anticholinergic agent are most often used in combination with albuterol for treatment emphysema and asthma.

A bulging of the tympanic membrane Bulging and air bubbles behind the TM represent OM with effusion.

The most reliable sign of otitis media with effusion is A bulging of the tympanic membrane B loss of tympanic membrane mobility C reddening of the tympanic membrane D air bubbles behind the tympanic membrane

glaucoma

Upon well check, your patient presents with ocular HTN. What is a major concern that the ocular HTN predisposes your patient to?

What are the main sx of nystagmus?

VERTIGO Oscillopsia BV Abnormal head positions

a 27-year-old male with intense nausea and vomiting that began yesterday. He states that he ran a 15-kilometer race in the morning and felt well while resting in a hammock afterward. However, when he rose from the hammock, he experienced two episodes of emesis accompanied by a sensation that the world was spinning around him. This lasted about one minute and self-resolved. He denies tinnitus or hearing changes, but he notes that he still feels slightly imbalanced. He has a past medical history of migraines, but he typically does not have nausea or vomiting with the headaches. At this visit, the patient's temperature is 98.5°F (36.9°C), blood pressure is 126/81 mmHg, pulse is 75/min, and respirations are 13/min. Cardiopulmonary exam is unremarkable. Cranial nerves are intact, and gross motor function and sensation are within normal limits. When the patient's head is turned to the right side and lowered quickly to the supine position, he claims that he feels "dizzy and nauseous." Nystagmus is noted in both eyes Dx: Tx:

Vertigo Antihistamines (Meclizine)

A patient is evaluated in the office with a red eye. The patient awoke with redness and a watery discharge from the eye. The eyelids were not matted together. Examination reveals a palpable preauricular node. What is the most likely diagnosis?

Viral conjunctivitis

A 6 year old patient comes with VERY water eyes after swimming all weekend. It started in one eye yesterday and now it is affecting both eyes. She keeps saying her eyes feel gritty. You look and they are a bit red. PE shows preauricular LAD and slit lamp shows punctate staining. Vision is normal. What is it? What is the MC cause? What is the treatment?

Viral conjunctivitis from adenovirus -supportive tx: compress, Olpatadine, Pheniramine-Naphazoline

Patient will present as → a 13-year-old boy with clear discharge from his nose for 2 days duration. He is also experiencing malaise. On nasal exam, the mucosa and turbinates appear edematous and red. Sinus palpation is nontender.

Viral rhinitis

Patient will present as → a 34-year-old previously healthy male with complaints of facial pressure and rhinorrhea for the past 3 day. You believe he has the "common cold". He reports 3 days of clear rhinorrhea. His temperature is 100.1 F (37.8 C), blood pressure is 120/70 mmHg, pulse is 85/min, and respirations are 15/min. The nasal exam reveals edematous turbinates and purulent discharge. The patient has facial tenderness with palpation over the involved sinus.

Viral sinusitis

Displaced otoliths

What causes BPPV?

optimize blood glucose referral to ophth VEGFi

What is the appropriate tx plan for proliferative diabetic retinopathy?

Central lesions: nystagmus will change direction, nonfatiguable, and can change directions with gaze Peripheral lesions: unidirectional, fatigueable, and can be inhibited by visual fixation.

What is the difference between nystagmus in patients with a central vs peripheral lesion?

P. aeurginosa must be covered with treatment options such as Cipro otic drops or neomycin/polymyxin drops

What is the most common pathogen for otitis externa and what is the general treatment options for it?

Inverting papilloma (nasal tumor)

What looks like a nasal polyp but is unilateral?

Streptococcus pneumoniae Haemophilus influenzae Moraxella catarrhalis

What three pathogens are most commonly associated with otitis media?

B Rinne air conduction greater than bone conduction; Weber lateralizes to right. This patient likely has an acoustic neuroma. Sensorineural hearing loss will result in air conduction greater than bone conduction and contralateral Weber

Which of the following is the likely pattern of testing on physical exam? A Rinne air conduction greater than bone conduction; Weber lateralizes to left. B Rinne air conduction greater than bone conduction; Weber lateralizes to right. C Rinne bone conduction greater than air conduction; Weber lateralizes to right. D Rinne bone conduction greater than air conduction; Weber lateralizes to left.

D Streptococcus pneumoniae

Which of the following is the most likely organism in a 2 year-old child with acute otitis media? A Staphylococcus aureus B Moraxella catarrhalis C Pseudomonas aeruginosa D Streptococcus pneumoniae

Perform an MRI to evaluate for a vestibular schwannoma

You 65 yo F patient presents with unilateral hearing loss and loss of speech discrimination. What your diagnostic plan and diff dx?

Central lesion of some kind

You perform a head thrust test on a patient complaining of vertigo and diplopia and the results are normal. What do you suspect?

Chronic OM Patient failed otic drops and move to 2nd line which is oral Cipro

Your 43 yo M patient is complaining of intermittent aural discharge that has been ongoing for past 3 weeks without relief with the previous prescribed ofloxacin ear drop. He denies any pain or fever just the annoying yellow discharge. His otoscopic exam shows a perforated TM but states that his sxs have not improved since initial treatment. What is his likely diagnosis and treatment based on pathogen?

"hot potato" voice, unresolved fever with medication, drooling, neck swelling, and dysphonia

Your 43 yo M patient was just diagnosed with Group A Strep via rapid antigen testing in clinic. You place the patient on Amoxicillin and tell him some red flags to look out for such as..... because they can be a sign of greater worsening infection.

Cholesteatoma is a AOM complication Refer to ENT due to possible needed for removal of granulomatous sac within ear

Your 43 yo patient has a history of frequent AOM but tends to not finish their abx when diagnosed. They present today with conductive hearing loss and painless otorrhea and a "pocket" in their TM. What is you diff dx and treatment plan as a primary care provider?

Peripheral lesion (automatically suggested due to unilateral hearing loss) Dix hallpike should have nonfatiguable horizontal nystagmus that can be inhibited by visual fixation

Your 45 yo M patient presents with vertigo and unilateral hearing loss. What do you suspect when performing a Dix hallpike exam? Diff dx?

Menieres has a triad of symptoms, tinnitus, hearing loss, and vertigo which is unilateral and RECURRENT. Labyrinthitis presents with the exact same symptoms except the vertigo is CONTINUOUS and includes nausea, vomiting, and lack of coordination when walking. But it is NONRECURRING and caused by a viral infection.

Your patient believes themselves to have Menieres disease upon a google search but you inform them they are WRONG and that they have labyrinthitis because her sxs have occured ONCE and was CONTINUOUS. How would you explain the difference between the two?

Your patients eyes are fixed on an object and when you turned their head quickly, there eyes did not remain fixed but were dragged off the target by the head turn. This implies a peripheral vestibular lesion to the side the head was turned.

Your patient is complaining of vertigo and you perform a head impulse test. The results are abnormal, what does this mean?

Neural tinnitus is due to muscle spasms in the middle ear causing a pulsatile ringing. Neural hearing loss is the least common type of hearing loss and is typically due to a neuroma or disease such a multiple sclerosis. It occurs when the auditory nerves are desensitized and the CNS does not receive neural hearing input.

Your patient is confused about what causes neural tinnitus and hearing loss. How do you explain it to them?

to rule out an intracranial mass lesion

Your patient presents with assymetrical sensorineural hearing loss and you want to perform an MRI. Why?

Central artery occlusion Treat within hours: lay pt flat, high flow O2, anticoagulants, diuretics

Your patient presents with painless monocular vision loss with a history of HTN and his ophthalmic exam shows box car segmentation with a pale optic disc. What is your treatment plan and diff dx?

A patient presents complaining of left eye discharge and eyes that were matted shut this morning. The patient denies changes in visual acuity, but states that he is afraid to put his contacts in. On physical examination you note erythematous conjunctivae and mucopurulent discharge of the left eye. The cornea is clear. Which of the following topical agents is the treatment of choice in this patient? A. Aminoglycoside (Tobrex) B. Olopatadine (Patanol) C. Cycloplegic D. Prednisolone acetate

a. Aminoglycoside

Which of the following is a risk factor for sialdenitis? a. Atropine b. migraines c. seizures d. chronic sinusitis

a. Atropine (anti-cholinergic, or diuretics- on quiz it was furosemide)

PT presents w/ sudden U/L vision loss. Fundoscopy reveals arteriolar narrowing that resemble box cars and perifoveal atrophy that resembles a cherry red spot. Which of the following is the dx? a. CRAO b. CRVO c. amaurosis fungax d. open angle glaucoma

a. CRAO (poor prognosis- urgent optho referral)

3 y/o F presents to ER w/ her mother w/ fever of 103.5, inability to swallow and mother noticed PT won't look upwards without screaming of pain. Lateral neck XR reveals gross enlargement of retropharyngeal space. Which tx is most appropriate? a. airway + Clindamycin b. broad spectrum abx c. I&D w/ suction d. hydration

a. airway + Clindamycin (or unasyn)

5 y/o presents w/ refractive ambylopia. Which tx is most appropriate? a. glasses b. eye patch c. surgery d. abx

a. glasses

4 y/o boy with PMH of cystic fibrosis presents with "bumps" in the nostrils B/L. Mom explains the PT has a runny nose, and appears to have a decreased sense of smell. Which of the following treatment options would be appropriate? a. intranasal glucocorticoids b. limit hot/spicy foods c. nasal decongestants d. diuretics

a. intranasal glucocorticoids (or surgery)

A 20 y/o develops redness and thick discharge in her L eye and c/o severe pain. PT admits to a new sexual partner 1 day ago, but believes he was a "safe" contact. The exudate reaccumulates minutes after being wiped away. After a few days of erythromycin ointment, no improvement is seen. PE reveals severe eyelid edema, chemosis and profuse purulent exudate. Which of the following treatments would be appropriate if your DFDX is gonorrheal conjunctivitis? a. oral azithromycin or IM ceftriaxone b. topical erythromycin opthalmic c. doxycycline d. warm compress

a. oral azithromycin or IM ceftriaxone

Which of the following is contraindicated with uncontrolled HTN? a. pseudofed b. amoxicillin c. augmentin d. doxycyline

a. pseudofed

Which of the following is not typically a cause of chronic dacrocystitis? a. strep infection b. systemic lupus erythematosus c. sarcoidosis d. dacryoliths e. repeated infections

a. strep infection

What will imaging of epiglottitis show? a. thumbprint sign b. steeple sign c. gross enlargement of retropharyngeal space d. cherry red epiglottis

a. thumbprint sign

_______ _______ can lead to dysesthesias around the face.

acoustic neuroma

A 10 year old girl comes into urgent care with her father after "she blew her nose so hard and now it won't stop bleeding on the right side." They just came back from a vacation in Las Vegas. What is it? What is the most common site? What is first line tx? back up Tx?

anterior epistaxis Kiesselbach plexus Direct pressure topical vasoconstrictors cautery, electrocauterization, silver nitrate, nasal packing + cephalexin or clindamycin

the source of most cases of epistaxis comes from what anatomic location?

anterior nasal septum-95 percent of epistaxis come from Kesselbach's plexus, which is a superficial, fragile group of arterioles and veins that are the most likely cause of nosebleeds. Five percent are posterior bleeds that originate along the sphenopalentine artery.

A boxer comes in to the ER with a giant, swollen, red ear after a match. He complains of a ringing in his ear and some slight hearing loss. What is it? What is the tx? What needs to be prevented?

auricular hematoma I&D in 7 days and then open surgical drainage. Cauliflower ear

7 y/o F presents to pediatrician w/ her mother c/o facial pain and a runny nose with thick discharge. Mother explains the PT has been appearing more tired than usual the last 2 weeks. PT has found no relief with intranasal corticosteroids, saline irrigation, or NSAIDS since her initial appointment 11 days ago. Which of the following is the next best step in treating this PT? a. Erythromycin b. Amoxicillin c. stronger corticosteroid dose d. XR of the nose

b. Amoxicillin

PT presents w/ sudden U/L vision loss. Fundoscopy reveals arteriolar narrowing that looks like blood and thunder, dilated veins, hemorrhages and edema. Which of the following is the dx? a. CRAO b. CRVO c. amaurosis fungax d. acute angle glaucoma

b. CRVO (good prognosis- f/u w/ thrombosis)

A 45-year-old male presents with purulent discharge from his right ear for three weeks. He states that despite being treated by his family doctor for an ear infection one month ago, the problem continues to get worse. Upon exam, you note purulent discharge in the ear canal, an erythemic tympanic membrane, and a possible perforation. What are the pathogens most likely to culture positive? a. strep pneumoniae b. pseudomonas c. E. coli d. candida albicans e. mycoplasma

b. CSOM The clinical vignette describes a chronic otitis media. Usually, this refers to a complication of acute otits media with perforation. Pathogens that culture from these infections are usually pseudomonas, proteus, or staphylococcus aureus.

40 y/o F presents to PCP c/o increase in tears and acute pain in the eyes. PE reveals a toxic-appearing PT, as well as erythematous, tender and edematous eyes. Which of the following is the next best step? a. abx b. CT scan c. Dix-Hallpike maneuver d. x ray

b. CT scan (or blood cultures)

10 y/o M presents to ER w/ eye pain that worsens when looking in different directions. PT c/o double vision and PE reveals a fever, as well as pain with extraocular movements. PMH includes bacterial rhinosinusitis. What is the next best step? a. admit and start IV vancomycin + ceftriaxone b. CT w/ contrast c. visual acuity test d. cycloplegic drops

b. CT w/ contrast (orbits... to confirm post-septal/orbital cellulitis)

20 y/o F presents to PCP w/ pain, redness, and swelling around new cartilage piercing. Which of the following treatments is appropriate? a. I&D w/ pressure bandage b. Ciprofloxacin c. Amoxicillin d. Topical erythromycin

b. Ciprofloxacin (perichondritis)

16 y/o F presents to PCP w/ severe fatigue, throat pain, fever and tenderness behind her ears and neck. She explains she went to urgent care, received a negative strep test and was told it was a virus and will subside on it's own, which it has not. Which of the following is the next best step? a. IM steroid injection b. EBV specific + reactive heterophile antibodies c. redo strep test d. culture e. lateral XR

b. EBV specific + reactive heterophile antibodies

PT presents w/ outer ear trauma/hematoma from being hit in the side of the head. How do you treat this PT? a. IV abx b. I&D'd w/ a pressure bandage applied after c. fluoroquinolones d. surgical decompression

b. I&D'd w/ a pressure bandage applied after

PT presents to new PCP for f/u on episodic vertigo, nausea and hearing loss. PT's prior PCP prescribed steroids to which the PT has responded well to. Which of the following diagnoses is this PT likely to have? a. Meniere's disease b. Labrynthitis c. Exostosis d. Acoustic neuroma

b. Labrynthitis (aka vestibular neuritis/neuronitis)

30 y/o F presents with recent onset of teary eyes, mild itchiness and watery discharge in both eyes. PE reveals preserved visual acuity, no corneal involvement, pupils reactive to light and large cobblestone papillae under eyelid. How would you treat this PT? a. steroids b. azelastine c. hygiene d. no tx needed

b. azelastine

30 y/o F w/ PMH of atopic dermatitis presents to PCP w/ purulent discharge from her R eye, pain, and a "gritty sensation" in her eye. PT denies use of contact lenses. Which initial treatment option is most appropriate? a. fluoroquinolones b. bacitrain- polymyxin b ointment c. oral azithromycin d. IM ceftriaxone

b. bacitrain- polymyxin b ointment (OR erythromycin ointment)

17 y/o M presents to ER after getting into a fight c/o diplopia and pain with looking up in his R eye. PE reveals exopthalmos, decreased visual acuity and a teardrop shaped pupil. You suspect a globe rupture, what is your next best step? a. urgent optho referral b. cover eye w/ eye shield c. send to OR immediately d. IV abx

b. cover eye w/ eye shield

Which of the following can cause excessive dryness of the eye? a. entropion b. ectropion c. internal chalazion d. corneal ulcer

b. ectropion

51 y/o M w/ PMH of hyperlipidemia and smoking presents to ER c/o vision loss that subsided momentarily. Medication list includes Atorvastatin and a baby Aspirin. Carotid doppler reveals amaurosis fungax. How do you manage this PT? a. change statin b. endarterectomy/carotid stent c. discharge- vision came back d. monitor this PT

b. endarterectomy/carotid stent

3 y/o M presents to pediatrician w/ recent onset of rhinorrhea, cough, and itching of his eyes. PE reveals infraorbital edema, transverse nasal crease, and cobblestoning of the posterior pharynx. His mother explains both of the PT's older siblings, as well as their father get these symptoms this time of year when going outside. Which of the following is the most effective treatment for this patient? a. allergen avoidance b. glucocorticoid nasal spray c. augmentin d. surgery

b. glucocorticoid nasal spray (most effective + oral claritin, nasal astelin, flonase, rhinocort, nasonex, etc.)

49 y/o F presents to UC w/ red eyes, flaking from eyelids, burning sensation in her eyes and blurry vision. Which is the best 1st step in treating this pt? a. cromolyn drops b. hygiene: warm compress/lid washing w/ baby soap c. erythromycin ointment immediately d. surgery

b. hygiene: warm compress/lid washing w/ baby soap (+ artificial tears; erythromycin if unresolved after 6 weeks)

40 y/o M presents to PCP c/o intermittent runny nose, post nasal drip and congestion. PT denies cough/sneezing. PT explains this usually occurs when eating spicy foods. Which of the following is the next best step? a. glucocorticoid nasal spray b. limit hot/spicy foods c. nasal decongestants d. afrin

b. limit hot/spicy foods

18 y/o F presents to PCP w/ painful sores in her mouth that have reoccured several times over the past few months but subside on their own. PT explains she is being tested for Chron's disease by her gastroenterologist. What would you offer this PT to relieve the pain? a. NSAIDs b. magic mouthwash c. wax d. lozenges

b. magic mouthwash

Which of the following is a cause of thrush? a. recent dental infection b. oral abx use c. otitis externa d. NSAID use

b. oral abx use

Which of the following epistaxis scenarios can be fatal? a. anterior bleed + statins b. posterior bleed + blood thinners c. nasal polyps + CF d. allergic rhinitis + flonase

b. posterior bleed + blood thinners

67 y/o M presents to PCP w/ U/L eye redness, pain that is worse when looking at light, and a gritty feeling in his L eye. PT wears contact lenses and noticed this pain on his way to work after putting in his lenses this morning. Which of the following is your first step in diagnosing this P? a. fluroscein stain b. visual acuity test c. erythromycin ointment d. cycloplegic drops for pain

b. visual acuity test

A patient comes in c/o a stuck shut, crusty eyelid when they wake up. Their eyes are red and purulent d/c is leaking out. They say their vision is normal. They do not wear contacts What is it ? What are the bugs? Tx?

bacterial conjuctivitis SA, SPNA, Hflu, Mcat, N-gonno, clam trach topical erythromycin

What is the prognosis of a patient with retinal detachment with macular detachment?

bad

Anterior epistaxis Direct pressure for 15 min Topical decongestant Anterior nasal packing

bleed at kiesselbach's plexus

PT presents w/ acute onset of eye pain w/ "halos" seen around light and watery eyes. PE reveals decreased visual acuity, sluggish pupil fixed in mid dilation, diffuse redness and eyeball firm and tender to palpation. Which treatment is most appropriate for this PT? a. Latanoprost topical b. Timolol c. Acetazolamide d. Laser therapy

c. Acetazolamide (+ apraclonidine or topical BB)

60 y/o M w/ PMH of DM and otitis externa presents w/ worsening of sx despite proper use of ear drops/topical abx. PE reveals purulent, foul smelling otorrhea. What is your next best step? a. debridement of the auditory canal b. oral abx c. CT scan of the head d. manage DM

c. CT scan of the head

87 y/o M presents to PCP w/ spinning sensation, especially when laying down in bed. PT denies any ear pain, hearing loss or tinnitus. How do you diagnose this PT? a. epley manuever b. pneumatic otoscope w/ insufflator bulb c. Dix-Hallpike manuever d. CT w/ contrast

c. Dix-Hallpike manuever

A 35 year-old patient presents with a sudden onset of fever, dysphonia, drooling, and difficulty drinking a few hours ago. Physical examination reveals a temperature of 102 degrees F. The patient appears ill and is sitting forward. Inspiratory retractions are noted and there is a soft stridor. Which of the following is the most likely diagnosis? A Angioedema B Foreign body aspiration C Epiglottitis D Bacterial pharyngitis

c. Epiglottitis

49 y/o M presents to PCP w/ U/L hearing loss, a buzzing sound in his ear and a tingling feeling on his face, both on the L side. PMH includes neurofibromatosis. Which of the following is the next step in diagnosing this PT? a. XR b. eye exam c. MRI brain d. CT brain

c. MRI brain (+ audiometry for acoustic neuroma)

Which of the following is caused by endolymphatic hydrops in the inner ears? a. Cholesteatoma b. BPPV c. Meniere disease d. Labrynthitis

c. Meniere disease

Which of the following can you not use cerumenolytics on? a. immunocompromised b. pediatric PT's c. PT's w/ TM perforation d. diabetics

c. PT's w/ TM perforation

Which of the following is diagnostic of infectious mononucleosis? a. Throat culture b. CT c. Reactive heterophile antibodies d. Splenomegaly

c. Reactive heterophile antibodies (+ EBV specific antibodies)

Which of the following should not be done for a tooth avulsion? a. dentist referral b. rinse the tooth with water c. bathe the tooth in alcohol d. doxycyline

c. bathe the tooth in alcohol

37 y/o F w/ PMH of lupus presents with a "deep boring pain" in her R eye that worsens when looking at the light. PE reveals violaceous hypermic patches beneath bulbar conjunctiva. Slit lamp reveals scleritis. How do you treat this PT? a. send to rheum to control lupus sx b. CT orbits c. corticosteroids + refer to optho d. surgery

c. corticosteroids + refer to optho

25 y/o M presents to ER after getting punched in the face. PE reveals periorbital ecchymosis with no injury to orbits, deviated nasal septum and a septal hematoma. After r/o other facial injuries, what is the next best step? a. put the nose back into place b. stop the bleeding c. drain the septal hematoma d. send to plastics

c. drain the septal hematoma

7 y/o F presents to pediatrician w/ her mother c/o facial pain and a runny nose with thick discharge. Mother explains the PT has been appearing more tired than usual the last 2 weeks. Which of the following is the next best step in treating this PT? a. Augmentin b. Amoxicillin c. intranasal corticosteroids + saline irrigation d. XR of the nose

c. intranasal corticosteroids + saline irrigation (+analgesics, decongestants)

Which of the following differentiates allergic rhinitis from nonallergic rhinitis? a. absence of rhinorrhea in allergic b. presence of nasal congestion in nonallergic c. no cough/sneezing in non-allergic d. they are the same

c. no cough/sneezing in non-allergic

Which of the following would solely yield cosmetic removal? a. pterygium b. acute dacrocystitis c. pinguecula d. a and c

c. pinguecula

Malignant OE is m/c caused by: a. group a strep b. s. pneumonieae c. pseudomonas aueruginosa d. h. influenza

c. pseudomonas aueruginosa

24 y/o F presents to PCP c/o nasal congestion x 1 month. PT explains she went to an urgent care center that gave her intranasal pseudophedrine which provided temporary relief for 1-2 weeks. However, PT explains this congestion has worsened over the past week. What is this referred to as? a. rhinosinusitis b. non-allergic rhinitis c. rhinitis medicamentosa d. incomplete treatment the 1st time

c. rhinitis medicamentosa

PT presents w/ pain and swelling near parotid gland. PE reveals some drainage of pus. Medication list includes Atropine and Albuterol. Which of the following treatments would you prescribe this PT? a. d/c anti-cholinergics b. add a diuretic c. sour candy d. abx

c. sour candy

70 y/o M w/PMH of DM presents to PCP c/o "colors just not looking as sharp" anymore. PE reveals an opacified lens. How do you treat this patient? a. monoclonal ab therapy b. laser tx c. surgery d. antiVEGE

c. surgery (cataracts)

How can you differentiate between viral and allergic conjunctivitis? a. same sx b. allergic responds to steroids c. viral has pre-auricular lymphadenopathy d. abnormal visual acuity in viral

c. viral has pre-auricular lymphadenopathy

What is the most common cause of conductive hearing loss?

cerumen impaction

acute HTN

cotton wool spots, retinal hemorrhages, retinal exudations

7 y/o F presents to pediatrician with eyelid pain, swelling and redness in her R eye. PE reveals no pain with EOM, no paralysis/weakness of ocular eye muscles, no bulging of the eye, unaffected VA and appropriate pupillary rxn to light. Which treatment is most appropriate? a. IV Vancomycin + Ceftriaxone b. Clindamycin + Amoxicillin c. Trimethoprim-sulfamethorazole d. B or C

d. B or C

Which of the following is not a cause of central vertigo? a. multiple sclerosis b. migraines c. cerebrovascular disease d. BPPV

d. BPPV (acoustic neuroma is the other one)

Which of the following treatments is not indicated for chlamydial conjunctivitis? a. Topical erythromycin opthalmic b. Azithryomycin 1g single dose c. Doxycycline 100mg bid x 14 days d. IM ceftriaxone

d. IM ceftriaxone (gonorrhea)

80 y/o African American PT w/ PMH of DMT2 and HTN presents w/ slow onset of painful peripheral vision loss. Which of the following treatments would be most effective for this PT? a. Acetazolomide b. Apraclonidine c. Prostaglandins + topical alpha 2 agonist d. Latanoprost + Timolol

d. Latanoprost + Timolol

30 y/o F presents w/ large external chalazion that has not resolved with warm compress. What is the next best step? a. incision and curretage b. glucocorticoid injection c. nothing d. a and b

d. a and b

30 y/o M presents to ER with a profuse nosebleed that has been going on for 1 hour. PT has been pinching the nares with no improvement. Before packing the nose, which step is most important? a. silver nitrate b. find out if PT is a cocaine user c. surgery d. a and b

d. a and b

Which of the following is not a typical cause of nasal polyps in adults? a. aspirin sensitivity b. chronic rhinosinusitis c. asthma d. chronic epistaxis

d. chronic epistaxis

Which of the following is not a risk factor for allergic rhinitis? a. male gender b. early abx use c. maternal smoking during 1st year of life d. cystic fibrosis

d. cystic fibrosis (seasonal is the other RF)

Which of the following is a risk factor for developing otitis externa? a. age b. family hx c. day care attendance d. exostoses e. smoking

d. exostoses

67 y/o M presents to PCP w/ U/L eye redness, pain that is worse when looking at light, and a gritty feeling in his L eye. PT wears contact lenses and noticed this pain on his way to work after putting in his lenses this morning. After a visual acuity test is performed and a corneal abrasion secondary to contact lenses is diagnosed, how do you treat this PT? a. trimethoprim-sulfamethorazole b. vancomycin c. erythromycin ointment d. fluoroquinolone

d. fluoroquinolone

80 y/o F presents to PCP with central vision loss. Which treatment is appropriate? a. systemic corticosteroids b. control HTN c. refer to optho d. monoclonal ab therapy

d. monoclonal ab therapy

5 y/o M presents to pediatrician w/ his father b/c of L eye itchiness x 2 days that has spread to both eyes as of this morning. PE reveals watery discharge and preauricular lymphadenopathy. Which of the following is part of your treatment plan? a. augmentin b. amoxicillin c. erythromycin ointment d. strict handwashing, no sharing of personal items, no school until the eye discharge stops.

d. strict handwashing, no sharing of personal items, no school until the eye discharge stops.

Which of the following serves as a pain barrier for dental carries until the PT can see their dentist? a. NSAIDs b. ABX c. removal of the tooth d. wax

d. wax

Herpetic whitlow is common in

dentists and health care workers exposed to infected secretions

Serous Otitis media is a problem caused by fluid in the middle ear. What will you see on otoscopy? When should you treat vs observe?

effusion w/ TM that is retracted or flat Hypomobility w/ insufflation Tympanostomy tubes for drainage for kids with hearing impairment, devel delay or specific conditions.

A patient comes in with a VERY hoarse voice and scratchy throat. PE reveals URI viral sx. What is it? bugs? Tx bugs if it were bacterial?

laryngitis Adenovirus, Rhinovirus, Influenza, RSV, Parainfluenza. self limited/supportive M. cat & Mycoplasma

A patient comes in with a painless, white patchy lesion that CANNOT be scraped. Hx of smoking and heavy ETOH drinking What is it? What test should you order? Tx?

leukoplakia Bx to r/o squamous carcinoma quit smoking/drinking cryotherapy, laser ablation surgical excision

What is the treatment for entropion and ectropion?

lube drops, moisture shield Surgery if bad

proliferative diabetic retinopathy

neovascularization, macular edema exudates/ ischemia, retinopathy

Meniere syndrome, labyrinthitis, vestibular neuronitis, and perilymphatic fistula are causes of?

peripheral vertigo.

What is the most common cause of sensorineural hearing loss?

presbycusis (due to aging )

Nonproliferative diabetic retinopathy

retinal hemorrhages, hard exudates, venous bleeding and retinal edema

What is the most common cause of viral rhinits?

rhinovirus

MC cause of viral rhinosinusitis

rhinovirus, influenza & parainfluenza.

An unvaccinated family comes into the emergency room with a very ill child. The child has difficulty breathing, is drool and is in a tripod position. You order an x-ray and it confirms epiglottitis. Cultures come back confirming the pathogen is H. Flu, what should you do for the unvaccinated parents and siblings not exhibiting symptoms?

rifampi, educate that this may cause pee to turn orange

A 2 year old child is brought in because "one of his eyes looks like it doesn't line up and stays inward." Hirschberg screen reveals an asymmetrical corneal light reflex. What is this? What is first line tx? what do we want to prevent?

strabismus patch normal eye amblyopia

What are the main tx of nystagmus?

symptomatic Acquired pendular: memantine and gabapentin Congenital pendular: memantine, gabapentin and soft contacts Jerk: dalfampridine (4-aminopyridine) (not for upbeat) Botox injections Prism lenses and solutions Surgery (mostly for congenital) tenotomy

A woman comes in c/o of eye pain and blurry vision. Her eyes are watery and light bothers her. She wears contacts and left them in while swimming in her pool a few days ago. staining shows pseudo-dendritic lesions. What do you expect to find when you cx her corneal scrapings? What is this? How is it treated

trophozoites Acanthamoeba Keratitis combo of biguanides and antiseptics.

What is rhinitis due to a noninfectious dilation of blood vessels?

vasomotor rhinitis

copper/ silver wiring AV nickeling

what are the first ocular clinical manifestations of HTN


Conjuntos de estudio relacionados

Anatomy - Upper Limb - BRS and Lippincotts

View Set

Biology 1030 Clemson Espinoza Exam 1

View Set

13. zabezpečování personálního obsazení hotelu

View Set

Chapter 6 & 7 Statistics Questions

View Set

Med Surg, Nervous System Study Questions

View Set

Delivery Types/Complications FINAL

View Set